You are on page 1of 114
IMINISTERUL EDUCATIE§1 INVATAMINTULUT Pt wir COST ,FOPNGL POH UEaR Pl, ALENT AEKAND Matematica Aritmetica e Algebra Manual peat cia © Via ‘FOTO BIDACTCA $1 PEDAGOSICN BUCURESM, 969, edaetr: po. afro Sis {Tarosditer fea tna IEC APITULAREA MATERIEN DIN CEASA A Yea Ta eae ce urpeasd, wom recapitule lteva ehestiun stuiate in lows Va Fle nomerela natural 15 sb 3. Exist mum naturel ast Best mimes onturols (4p 2 exitt numtral natural anf Sot Inmsini en 2 4 obinem pe Hs Spanem eu omar watara Bride pe 18 Se tal spore ch 182 re oh 18 coe di Xm Bui 18. Nomdral rotor 18 eee tn mealpla el a 2 Satie ela eitire fo yrs Wi Teco el uetren-vataral sll tne nsullindut on 4 olfinem po 19. Spunemn ett divide of Dette. Wun Esenpe Dele 5 ‘Namaral palaral {ay este nomér ri. Cal med sure dvizr comin (peeaeetat emma.) a Wai $9 8 este" Varn eri weenta all 5, 16) =2, Se ste of dood seriom 15-=3-5 si 1822-9, unde 2, 3.5 Hin, nuloee primey stunot axmmd.e, ol ta 40 18 eile. prodal [Mcbrior riot tontns Tut AB'qf 4B fesare factor priv fi tat 9 Tingura tcl pera ose mel ied cw care fgutvash(n Uo #18, In enon! nomenon natorale 6 #95. avem 6 2-2, 25 =: Namencis naturale G39 am ao Taetrt primi come. Singral ler ‘ivior cont ests 1, Cl mt mare divi eau all © 190 ets trelor naturale ¢are maw factor pram eomani, Tatura i, eal mat mare Sivienr co ison comun a for eit 1 frst do maveatiespenten clase a V0 ax fost dat, fay demonstrate, urmatoatla eoremes ox prime ine le dai 1. Dac un miimdr natural ete dotabil en daa numere naturale Peseupa, 2 ate diviiil ew 2 gh eu 9 Dar 12 esto dvi gi 2 Morena impinge teorema Smaieteu rst: Oriana orf nunas atures Os undeb #0. exst dowd numer naira ohn monitorspeto et 3 rx ae nat amb tr rch Wnmersle 9 gi r determinate de acre cont, stat nice Dori r= 0, atone a wp divide eu Uy 0 Duck PU atanet @ nu Wo divide eu &, 2 6, Bsempla. 181 —25-7 +6, 6-225, Deoaree 6 4 0, rons ob 25 a divide. po A, nai mie mali comm (pesoartat eom.ma.m.c) al Ia 15 4 ental nl fe mie natura dere de zero care eto uli ‘nia 13418, ete , Wom srk accucta stl 15, 15) = a dad serio 15 =3- 9 18 ru a 1396 18, feare fir prim ft ‘Ninel astra stam a afoot pein ea puns gucomim, O21 mal me matipa enn a ‘steels le num nator, dere defer, care eas mala mua al iy fy adicd eae 1 Voth serie acensta stl Ly 1] — 1. ete st nontme 4. Si ge oootaen 4) 25-40; b) a0 f99 2 49. 4 a) 15946 + 9.887; e) 19910 + 8080. = 5 6) 20000 + 6065 3 on 5) Ss aoa § toot ) ict i peo oo 1) Num 15) Sa eal [8 fers00 | — $) Nutmaral 47271 ete Url 410) Numbeul 4570 esto divi eu 2 Af Numara {485 este erie 1} Nimara 2) Etectoti: iW 240m, by 43.444 — Im) — ae) Seat $550 000 — 19 06, 1) ton — a ote fo) sn. 1005 0) 2 ictus int divin nomial parol 13h ata diel cu al 63 G24 ate divi Gea 13903 ate divi fe ompletee tall un Ba | Da | Ne | Ds | Bs 40 10005 A) 20-0 Ky ke (Bsn, ie — 666 no Dar ai mmdrulad 18? 2 Dar Der ew 1 2 afl toote aumerela de forma FR" divisible en 190) 1b) Si ao fle tonto numersie deforma {fa dviibile ew 6) 53 all Looe nuoerle so forms (8 dvi eu 3 4) Sige alle toate nunerle de forina TE divabile ou ) $6 a0 alle tonto numerle ds fort 5 TSr diviibile en 4 1) Sb ae alle toate nutnerle de forma Ayr aiviibile ew 1 ) Si so ale tnte meres de forma 20 pup dive e 3 17) ‘Gar eate oot mal mare divizor commun (etude) a une relor 8 10 18) Cat ete co ral mie ruin com fesman.m..) a) mumere tors 2; 3542 Dar sl mario 4,02 109 Dar al aucnceon! 4480 97 1,88 te tlle canst al anmerlor Sys 3; 74, by At 3: 0, 20) Si_ rw afl comma. lumen a) 38, 74: b) 20 U0; 4 SOD) 310, S00; 80.) 19051 6005 aan, aay tae oy 2-104. 4a. @ fortarde 2 a, capri ans f) 2:80:10 we tie Lia ote aan aon tee £2) Sa co dri lel rsa np avmiiat 618 (8D ts mumarul 2000, acl vontecsa' (fet ! 2 OPEAATU OU SUOMLRE RATIONALE g CU PRACTHE AECIMALE fears atthe Ad fae wrmatoarcle. opera ea, He turers rationale refjonele ote simérul ralional nota tear so obine ‘tel: Se adve fractile 2 yi 2 la numitor cotiun, Pentru acoasta 1 fe calevlaza eiman.m.e. al numlorlor 3 gi 2, anv (3, 7) ‘poi frstia ~ se ampli eu ctl en 23 ar acta aryl otalHnte 24g coer eat Soden. Fin moarsl rlionsle 7 9 atonal? sf ruminal atonal 2 cate numaralrafioal ti cose obfine ate. Sau fractite 2 gi ta Inprivea. Pin numescle rajonsle © gi mere raion Samir rtional 2 ampltck on etal intro ph 4 Se obin face jonalo 912. Prods aestoraumers guar rajional nett on £2 care we obine ante sae stele Cl ntre vest nue ate oomural rational oot ew = 2 gi are se obtine tnmulhindasl pe eu fnversl fai 2 care este nomlra rafinal Trmalien, Fi fei exccnale 17. 44,2. Dou ral sc ‘aie te intunieee ca dona namere oatarale, raul and ata Five dnpa veg ete ere dup cigula sv ijprovnt cela dou lee fi rectmele. Desi al ce se abae atune clad 4» cunoate of seri sob forma rationale de forma Geer etn eke a pedis. Be ttle secinale 1748 94.5. Chat tmp an pea Pacis reese ae (oat at oem freci seeale 1743 prin tralia secimsls ‘15's tine sll (0 eitota fare 9 Seam rasta aeoimal 12,007 eo et ‘6n frac neimale foo watts opeafi Aivnere. Vie teseileseeimale 45007 q1 120452. Actste dowd wales adun en dou ‘in eal dowd frat a vigul numero naturale: dupa ee au feet it meimale aioe coneapunda, Decl In ambele fret mevimale 7.43 i 1,5 vigula 9 fost mutata ta Aieapia poste o rua, ponte tscsfotna tect retold i huiniral natora 1. pm caleulle sf eu la tunese ta india vingula In resulted indie cx ia Aa ving tn 17 Hua ane antek aerial 174 prin fnetn serial este vein eocinale 14,42 cave aie 9 fneie zccimal Ta THe tools zeimale 0.80 4b. Avem: Us mum natal dando cu o tfc toca consierind Vingul Sota dupa ulti ei a se ara considera xen co aE ects ee BE [15 . trait socal cua eal all roar 185 faa ine Sdn, ft ima 004 wi 2807 i eine oe pice ain Tent ea vigulle din ule Wow fri mcimaly sn conspands. Deets (ltl impart taste selma 0.288 prin tralia evimall 15 8) Bieetnt neste 0 foelie seca fit, el ett o facie seoimall perdi se aatniie 676,105: 609.5: 6190948 —290.545 O.(42) eu percade £2 a) bios 141 can? — 3090080; xc gt rome 1p) 0.2- 100; 6) 04-1000; 8) 24-58: «) 9007-05, 2 g) 40,8" 20,5; h) 24 10055 1) 2 08-4 200; is I) 2,4" 20 00,2 1) St sep ay itioe foci 8 1 000; «9 92: 4005 2) So se efvctuen Toy my Li033: 1) S001: 0) 02:0 si 82 Od) 16 74683 Rom re urnivoarle lane! cu dou seimale 15122367023 1) St 12) Sh v0 acters te #) 090 M0 10; by AD0ON. : 4007 — 4002, 9 6905 28. — 990 0; b1 4-284. 100-1486 + 10-484 ae) 1 off: of fate o0eoy-02 : area evans ie Bite ei on sin; A) Am0- 14308 4) 4000.09. Ey 0 ies 15) St w ete a) (os42 6) 8-4 008 (0,000.4 ) 1049) + 2465) + 0428) (4 + nya 16) Sis efvetuem 7a} Leeeea| f) 06 FF O02. 10%, bY 1) 1002" 10-0 108-0 3448-(o8m1—98139) a_i meearin ven Dy 1 2A 4 12.46 008 421 12) $4 te alle raloaten de adevte a floxéreladinto urmétoarelo pai 1B) 9 10" < 40%; b) 2020000 = 408. 200; 10° <9,900- 40k; 18) Se considers muni spit Am(h 2,3, 4; Bit, 2; C= 10,71; D= 10, 8 OF RAPOARTE §1 PROPORTIL 1. Si afte valonten da adevir © vem@toatelor propor htcas IEC occ ACH, 44a By HOS D; A) AB. 1. RaroKr, TI. $4 4 aft a4 Bb) AUC; 0) APB; A COD: @ BNC DCD sD—¢ io numero noturale 6 gi 3. Vewm st sim do ete or este mai 19) f4 consider motes rare numdral 6 deci mnimsrol 3. Pentry aevasta fom inprties a rent TMU er Consatam ef tumarul 6 este de dont ori mai mare dest Oe come ees ‘St be roprenintaflovare dine geste multi enumerod eens peer ce mira 2. Sivek % este faportal dates nomial 6 si anmaral Putem sie” — 2, Nomirol 2 ae mumesteoaloaren rape ©) Ste = 005 d) 2 =; Un aft exomplu: roportal dine nui 3 gi numa 9 esto ene wo, Nana ‘Termeni portal pot 6 mumererafonas on in exempta weston 21) Stn cf a este nombe rai aia, ao elena a) a4 4a; By gpk. onal nenegtiv pcm este num pat ts dye @) s9 Raportuldintee mumaral 2 gi mumaral 4 esty 5. Arem ® 2, Dect & esto valoaenraportalu dintre merle 2 al ok momtral A, iferit de fn general, nepoctal dint ao reco, se noteas cu. So maj spuno ct 4 este raportal dintre ume tele e sib sau ch % este raportal dint a xi no data incite tn diterita expriminiraprted mnierlr en valor sesstin. So spine de exernps Raportl intro nmodral By aumtrald este Obeervaic. Un raport ara o valor aumai dick termenal do rub Tina de faefie ete dart da zero. ovseaqu co ctvon x xtoens marionatn Oeratile on eltrk de nsmere rafonale fad operail en Galo de capoarte, le vom numl pera rapoort Consider 5m 2, Duck d= cgi inn yea. de fptal ob Daed e, hd lot mumere selonale, jar BA 0, a # 0, aundl bee dba qUMMe te, a ov 4m wikimal eux avind ¢1 ¢ 0, Se observa ca cele pate opeafi ev eltari de mumere rofionale tat formal identice eu cele patra eperai eu numers rfionle Simolsl de fomultive se omits or de cite or ete posi ea ta anil produseor es iatervin in srierile do mai taint Si vestoim foptul o8 evom 2 «pei a, Di in into di Am verifies oh pentru qlitatea fs caste tok tered 1 We onlne ek LA Go std da gE, 6 #0, fete raportal lunginilor ail douk aegmente, mihueale ci, seseay Unilste' de masusl enee ea mate. Pentru a pune tn evkian Unitatea de nidsure cure Ia eazal de fafa este mele, raporta Heab forma 25 9, do ace, vom admis 4) Do cite oi este ral. mare Tungimes. dott limes? ox : ae 4) Cave eto portal dato atime | angie? 8) Do ee on ere mal id nea doc. agin (Gacai on down) 4) Longines ted bool do sent ete de 4 cm, Sn tong ice ‘este de 6 dm, Si se alle raportul diatre lungimea primei Soon gies 0 do dc © cgalitate de feat acosta va fl exprimatd stl SS Sere ors ee pier Minstoral perth e cowade into nar af nasa 3) Raperta ‘oe 1 Hstmol 1 ftom va aft 2 7 de ps ar i leva aft 21 deaps. Do cite or ete al mare volume do pi din paul Yok ect yume de op din al dott yas? me Fed portal 2 admitom oa 12. Dect vlumal de ep 9 9 din primal vas este de 9 oF mai mare deet SI doen va. dc SS ica b= 4, 64 oe culeubern volar de apt din 48) So io cd roportsldintee «este egal eu 3, i raportl dint Gi onto opal cu 6 Si ne caloulee reportal diate Bremplal 2 Un yom este iat do 2m, iar un ata ext tat do 4) lamaginen unui droplungh este do tet ori mak mare deca 4m, Det ori ste lea primal pon mat mit dete ljunee, men #8 cel det da? a) Care este rapertal diate Jim $i lingne? acind reports], coostatm oh 42 2, Dect pimol pom 1) Ge trate din Tongine repreiat lien? tad ou lod di Bijan etary le cn inti ae : Sk to ofl Fc roportal 2%, constatam ot 2 f) Raportal intro latin ¢l longin fimen pom tai de 2 m ete tacia Edin tealtinen pomali 3) Rapeta ate tango laine. Vom spans int Ynalt de 4 a 2: PROVONTIE.PROPRIETATE FUNDAMENEALA A PRononTIEE Paportal mdeurilr @ dowd mdrini, mdourate eu acousi unitate de ‘mdcart din enre prima ete mat mare doch a dona, ne aad de tea rina nisi ete mat mare date da bau dees oi a doa masarh Find. dtm dons. seqmen Ee mat mics det prima, Pipe eslieerent wrt avind Inapimen de 10 em th lta evind Tooginen de Brn, raportel inte fongimen primal Segment Hi longanea celui deal dalea erent eo 1) Lngimen unui dreptonghi este de 6 m gi ajimea de? ‘ aed tungimen tectruin din osle douk segmente este miaveatt ca 4) Gu cli centimetei este mat mare Tungimen destt limes? S.unitate de misurd egal ov, 2 em at 1) Gare este paportl di , nay lupgime’ prt engines eune? sent este egal ou 5/2 om lar a eaul deal dolea ete egal 3-2 0m i del eapontl ttre Ragin pin segs lngimen celui desl dale vpient. ete Cale dow rapoitte sot pate pontrn et exprint acess lacy vine raportal inte, hngimie eelor: doua eegmente date, yale tatea" elon dou poor ao sei 0 atte do ealtate © vom nimi propor Det. Proporta exe ealite te dou repo Din proprintatile rapoartler, sabilite «x gona studio opm rogioe dt cltrt ae abatere atta operat eae Tooth nue ‘pera eu rapoartey it vanut el : tn care varie expres In prin et apne eh roportal 4 fost bi dn por tol prin emplifiare one Jn carat al dots, se spane ed raportal ® a fos ebinat ia raportul 2 prin simpifare eu e proper Bsemph, ta oe de puto eto 0 pooportin ferment propos ‘Orce propa are patra tri © properties numese Basingle. int propor ‘Conituesempla, Nom ae 9 mamaria simul rapt into propio. | porte ar Muuitan weestal rapt ct 2 mii nL enamine dest aor sl aneiproporti wate al trelloa termen al proporfe, fat atonal telaia! rapt steal patra tren. a proper Pala termon ial ptraten tame al wet propor se nume arent, iat a dolen trtnen Wi At lle eemen a one propuril Exempla, Tn proportia Sh proper Jp gral, nem propiten fotonentl o pops Demonstraliaacstelproprietifi este urudlewtea, ‘ie 0 pro pore, urlesre ar ft aceata Din doula eftlai abt a =Br, ¢ = dr. Dosh af = br, bo = dr Aviad bed = bar aver a be lind to dowd rapoart gh oo ataigte of acest eapoarta formeazi o proports,verifitnd et proprntntea! Find date dowd expo be, dooarwce este adevinat portiedaot ad In alevir, dea ed =e, avind d # 0, stn nitia impair, daducem a ~ Com 54 0, fnted seams do detiniin impart re Exempl. Si se enentonn doe ntmerele 2, 4, 4140 pot fi tox seni ne pepe i Caealnd toute produsele i enre Satori sat dowd tin nymarele 2,45 san 10, an din foctot find 2 4 comparing rca, prods ‘ex produsul elellte dosa mimere, obfiaem: 3-4 5. Wi Tot 8" 50; 2-5 # 4-40, ponte GA 10 fs 2°10 — uti gallate rezula et dumerste dats pot fi trmeni port 1) Fie proporfia $=. $tind et af 10, a8 v enleulze 40 2e se ealeulas a>, 2) fe atiec 4 3) Pla proportin Stlind of @ =, 88 te calsulezs d+ 6, 4) Fie proportin 4m, Su ne enleleae 5 + £4 5) Stind ob LE i ch o-b-=21, sh op caliubse 6) Se iio ea + Si ve caleuese 1000 ab 4) Stiind of G+ 2 = 5y, sh co alle 6) Dindnse 00-2002 4, x v0 ale ©) Sestincs Sep m4. y, Sé ancaleulee 2 10) Se tie c& dae fammlyim nomiral « ev wbtnem sceley rau fat ea afuned en famutfim nnosrol bx 8. Sh clean 41) Stind ed 2 din rmanul «este gat cu & din mumaral 8, si alle raportal dintre ib {APLAR ONCT TEMEN STVENOSCET AL ENT ROPORTEL tee fundamentali a proporici ne arth ch pentra proportia 2—-5 vem ad «be ‘Am mal artat cS rapoartele 2+ forienzi 0 proporie dict ted = be, Adicts dach af =e si ¥ 0, d+ 0, tunel 2 Datori opal tem determina oF Ain cei Geli Inparyrd tau tn baza definiie impitini Io be prin o, dock «#0, Doct tmpérjim pe od lab, fn carl cid B¥ 0, chinem fan dac8 mpértim pe ad ls im eazul ind e+ 0, remit , Canute In core determinim pe a prin «= sau determinim ped prin d=" pot i exprimate atl: cit esta 22 Exempla. Fle proporti vom y=Sz = 2) Sind eh 4- 0 03: oh wale 4) Son ch La 3- 5. St scales 45 8 pte forma o proporie? Dae eu mime 4) 6» nm 28) See afte din ee false 38) Sa seo 497) Sie alle din 38) Si ae fle din: 1 = 29) Si w ale g. 40) Si 9 alle Pda: p mE, 41) St ve alle 5 ain: p =~. 48) Roportul diate pref unui erion gi preful unui caiet este ep 4) Pretal eriomulat ete mai tare sow mat mie tala? De elt et ‘by acs prejl ereionull este de 2s, 8 43) Raportl dintee pre voi edt gi pretal unui stlou ack still costs 90 Ii #8 ve alle elt eostd carton 44) Taportul a doud numere naturale exte shat tpl ene leet cost eat iar cel ma se dintre de este 12. Si te afl 48) Rportal & dovk nome dint ele 160, Sie alle claalt, numer 4 rnorongi: neniyare noon beni ce acts remvnst i ei propor = nef P "nd day patra ten Inclt a #0, 04 0, 0 0, eae 0, objinem tot peoparit prin urmae Se schimd mesit inte ei. Dinproportia = se cbtine pro i exromi tire ei Din proportia = a0 obline peo Din proportia = oe obtine propo. Cale trek proponttoihtnste din prima se numeso propa dnt i proprtei = sp capita rvtal on ele teh pocede, avind ca punet de plocre proportia deta Bxempla. Din proportint) 4% eoath ma urmbtoase propo caro ax ea terme termenllproportet dete prin achinorea extemilortntre et tm ptoporia {5 5. oni invrsen pmol prop! = prin echiboren seston itt o propor 2 prin schimbates extrmilor Inte of ta proportia & 6) 2 prin schimbaen srs ie ota proper 7. ‘oronTT Dem 9 CO ALT nent ‘i conshdenim propor = £. Putom seri Am obtinut o roportiadevivata din prim Se'poate arate oa di popor(ie objine urmstoarele propor derivate Ze unde f+ 0, atuneicind f se gieeste le nomitorul cel atin al uni aport, So pot objine, de asehiones, womnatoarele propentit derivate phe Rae gh ise & at Osa unde f #0 Si doducem a ota a) fee, Am véaut ci daci avem proportia $=, atuyelavem gi egal ‘tates oa = te, jar daca avem egulitaton ad - bein eure 6 # 0, d ¥ 0, ‘tune’ avem proportia * — 8 famuttim ambi membri ai eealitititet = Be on f, #0. Avom (ad) =e} feeau ce we pus vere cate fhe oP vom deci propontia = 5. Des din propria m5 am obnat, Proporta, 5 ete 0 proportic deriva din Proporta = Provertis 2 Intr-oe mad agemsanso EN (et as toto proponti Y “ah 1 poate proved gi taal proportiloe i derivate din proportia ‘ obfin to modul urmator. Senotear eur valoareecomeni a rapoar- Alor $ gi $. Desi £ mr, £ . Tint wama de doin etki, obtinem ¢ Br, ed, far prin adnate membru oy memivo a eestor dons eyaltay even eye thse. Dark bed # otunsi $5 or. Desk din Pa Sg b+ de 0 rowla Soisind membre oo mombra eglititle @ =r, ¢. dr objinom a= c= (b= dy. Duck b= d+ 0, atone Pr Dei din = 5 9 44-0 objinem: Bxewple. Fie proporio 32. Tnind seams’ de faylal ob Baler doe tesla saeet epentecettry ’ mee dla wrmatoors prop “2A neninit eprrtie pope it tr eae wf dem site inl ee Des 640 dn, ma, f=, in ee ono tettt, Setth Fetal Dp unde obfionm rape, ty ot a sel nomitori rapartlor eestor jwopori int. dirt. do aro Dis £atts, 4, deduce i £408!) do unde at ‘iter do zoo, emits * Ack nomitri ropoarteor ace Ae chat Fsempln, Find ta propria #=2 ave qt propore Accste propor find respetiv 8 =f. Avem proporti, deoseece raponrlee din sembral fli propor we oti pin ampifiarwa cu 2 a rs ‘mombral al dil al aeator propa, cu exceptn proportiei 2— 2, ‘nde rapontl din oeaibra Sas ap obfine prin atopliearoa eu 3 roportll din tetrad al dln : Problane rexavate 1) Soma a dow numer este egal on 48, iar raportal lor este 2. Si safle numer a Messisary. Kio iy cele doud numere. Aver: z+y= 18 Stim ch 22. Apion eteva dn oe oa be Bye 0 logiturd ch, proportile derivate Seriom 21, Deci 824, de onde y= Dar et y= 66 Atlin pos 2 alte y 8 — 20 = 18. Pater ala po x gi din: $= 2 ‘avon: 8 3% = 1p 41) Sum a dovt numer este 00, iar raportl lor 2) Se gtie ot Potam proved ato: Stic ) = De ssl ded Aven Dei 2S 4) Un ceva depus 6, 3 portal dntee sue Prima met , 2 ous tun este eg) en 2 {Cn doua uni dele soma do 20 tn. ‘in prima Tapa i sma depued in (Cb sum @-depue elval in foeae Ancoikin gi po = wa? 8) Limes unui dreptonghi ete dn Inga 8) Sc ale apr dite Iegine gine B) Si be fle repr) dite perinersie Noid uml eunetints retire 0 propo dvcaty pe TB 6) & comidert tingid ADC Sgt ty De ay, BE (4c cae Si wate of dock area OW th aoe oe A dos matt octal et peak 4) ave : ot Din = 2 obsinem 2 =! 5. $e DE Raroseere agus Inlocuim in apt $ at fe oy rota & an eater des Doll gre Hopirla Lm, Deed report 9 ampliicim ox fe otis Co £40, obsinem proportis i ast ‘i 77 pum cf avem un gi do rep eames 2=4= 8 eto un gi do ropogrs ple Un alt gi de rapoarte egle ent urmstoral ck womitor slat di taerece dock rete valour comsnd a ropoorteloe ©. £1 © orem Poe ae tee eae ee + eu condifin co da +47 0 Tinind seame do foptul cd Say cob Wa renal Exempla. 2 = rf aumea e Bram mt ail do ae de pana spoon dn ob fom © opr 1) Sting os 1 ah ve of 2) Fe St ce caeulers 4 4. 3) Deck 2h wit 1 oe calouere Si sa caleylems +d 8) 5tind ct dbotepena, Icha tbtent, simalle eb yic erare peotru veificagea insur mine ennogtnte de bax 1) Namirul 3240 este divsiil ov 2? Dar ew 5? Dar ev 102 Dar cu 5a ctsenea nit 0192-4 924431005 iy 2 uy, 4) Mayoral dinre ims. tunginee onukdroptanghi ete 2s aera ponten pees olimpiadsler yi allor eoneurcur 1 Gare sit nomercte naturale alc Fi egal ew lf note 4) Fie a= 1-2-3...99- 400 ale 72? 810, Cove este reste impintrt i 5) Lites ut depth ete att ow 2 din perinotrl a aportl diate semiperimetty 4 lime 85 0 fle reportal dint Tifime a hregtay ©) Daca tangimet oestut drepionahieete ea i6 i mai mare Owed Si vo ealculee 4) Sk se ale» dia PROPORTION ALITATE DIRECTX. PROPORTIONALITATE INVERS 6. Phorowyroxauaare pmméert Fic urmatorul yir do rapoarteegale Raposrtle din acest gf de repourteegsle au o vars comand, care o putem exprima prin = sau 25. Spunom et va stehite 9, epertonaiae diets ise rn (9,25; 125g 2,40, 604 ‘rin rier ial de raponre oat de a, coe gener ir dat malt fate de namere eal 0 proponionatae acta poats forma un sir de voperta ak fe ae et mele numero aoa a feo se itr malin namtortarrapearicer et fe eek git Notind cur valotes comund a rapoertaor distron ge do rae oar eal, difevite de aero, gu cricare din raoartle din aces. On 6159 0, Desis He de raporte eae, cynam £ rin naive clement ane moti fede namnre, diferie Hxrmp. Fe limen (2.3 are Hear aoa ep tin namie de stile de late, Diayece 9 trl de lapte ests SG atone! ullnea cs, 7, 0SUL Ceca me capone So Be fsto.un. se de ropoarte Sind a fe, ealares comunk a acestor rapoart (oe in ll exprima ustal el Miele de iu 2. Puovonriosatrrare rovnist Fic wimatoral gir do prodase egote Prudele din acon Hanelie incrsd intro wtlfoe, firlut de produseegele de ‘gale si valoare comund, pe 4,8) i €25 30, 100) prin deren, Inver dace poate forma wn it de prodsse Seas IE autfel tilt mulginen primar fort ai producls a feng diced imi, ior maltimea edotaltjudrs ak produto oj ‘naling Dotind ea + valour cumusa « prduslor diteson gir do pas aes divte nor fon ad ‘lia pruiele din abst pe de produso eye, obtinein ob = reau a =, deounece ¥ 0, Des Prin impair ai name det, difert de zo, cu denenicle o aine Exeinla. Fie mules (10, 1,2 in ear evare nme epi Uagimen in tier d'unut dreptanghi a carat aris ete me Atune! in multinea 280, 208,100) fecare numar orpricck heen in metri dreptunghiulol consderat Tutte see deca nee © proporfionalitate vets deosrese 10+ 29) = 16. 200 4 90-440 fete un vi de produse egale valosten comuns a acestor prodan find 2800, ecot'ce nm expriai ria unul deeptnead 4 ROUULA pe mmst stu Bar considers probleme in eave sntorsin dous aultini de ete MOU nurse Unite care esiss 0 propuriousitteUnveta sao roportionalitate versa ir unut diy'nvnerle-uuel ulin este Pracedenl care se folovte: pentru determivarea numdala nec ose dint din do mn de ceded fuamere tir care exist proporionaliae dnc sah a proporional late incre se numeye rege de bed simple Eemplul 1, 151m de stole costa 4230 lei, Cit coma 27a dia Formm, mai inti, multines (15, 27) n care numer exprimk ‘mote desta, Forman, upel, molfinen {1 25), x) in eave, nuherae xprima sumele In fl penta 15.m do. Mol "speci. 27 mdi cect oof Totre-acete doul sultimi exits” proportionate Aires, deoarce 1H, 2 cnt egale, valeurs lor co ‘muna find costal in lei wna mele din stofa cousidorats. Dei Rerats 21 = 2214. Decl 7 m din sola con iowa costs $214 te Se spunp 6 rezolvarea problaméi de mai inint, prin etieroa roportict [2 facut prin mao Iu toc de a serie proportia indicat are ae cilete asiel: Dac, 15 m alolé cos 1 20 teh atunel 27 m Din seers considera esi ald se poate ere gi proporia #2 4a aeolsi remutat arent. Dacd 48'm stoi conts'T 200 ely etunl fm sy asa oth cont lxallé cl 27 m din. souayl stata contd a” Ate raoniment, on date mata 1280 dete 1297 201 xranpla 2, Un ek are 0 moneda Ie, Cite moneda § te poste eipata tn schimbul muni’ pe eure 0 are vali monogr. Foroan pes, muljmen 0,3] In cape Nomen fxprins naira wneril de Wt, rape a tnoneor de> Tathe soete dows milfs exith o poopor[onattate myer de eee prod 10, Sain egos; valonten lor comand id dea Sind a ii Dect Result «= 22! 6, Dei 6 monedi a 6 Ie fae Lot atita ct 40 awonedt 2 gultath 3°10 =r, care pn a provi dia In lac 03 il, Din ah rome fora a din ®! monosi a5 ui. Ace eafionament, ca fet male raltewit Ja wnat, 29 ayeed sub forma vemétoure heat 3. 10 rie Aeei 1 6 1) 50 do ervione cost 100 bei Ci costs 7 erioane de acta fel? 2) K hig de moro cost 32 it conti 7 hg de meré de aclag fel? 8) 5 kg de aabar contd 70 le. Ch costa 8 hg de mie? 4) O ront face 100 dovotati a dow minnte Cite rotalit fae oat 1) Peatra a ralza 50 edie son consumaa 4 ky hive, Ua hit ta consuina penton a rnin 30 ees do dela fe? 6) Din Oke eafon Vendo so obtin 5 kg eafin prt. Din clte kilo rams de eafen verde ee cotin OO kg eelea prijta 1) Din 80 eg grin so obfin 40 leg fins. in elt Klogranse do gia ‘yin hg de tna 8% Dowd mobile in mise recto. oniforma parcurg_aoseas stanyi, Raportal viteaslr este i Sto celeueesraportal Interealoie de inp In cave este paisursa aceasta distant 9) sot amin rao in 6 oe. In ce or erin Tura 10) 6 monetaritarmjnd oIocrare ip 8 or. Ta ete one tarmnind Inerae 1) Un mobil s¢ mises uniform rectiinin on viters de 40 kam[h gt pareurge o anusits diana in 2h In et timp va. penomae oat mobil aoeud ditonte, deo vitesse va ff deb Tank? 19) 4 robinete pot umple vn rezevor in 6 oe. Ta ft tmp pot ample Slat! remmvor 2 rolineta? (Se prespune ct tute fobinetce tir ala debt) 18) 9 robineto pot ome va besin in 4. ore Ta oft tip vor putea 2 Umpleecalay borin 8 vobinete?(Rebinwtale at celal debit) 1) Two twectoare pot ara o soprefate ageicolé tn 200 ore. In ft Up Yar putea are scoeat supratata 1a tastoar? 15) Pentsu s°vops an ih ea flosit {ig vopsea. Cte Klograme Fis eopuee AE nati Sevres w Sepnimu on oa ‘de dont oni mal mare dent's primal 36) Un basin dé forma cs enb at ample ou apf, cn ajtoral nei pomp, in 45 rinse. Tn st Chap se va Umaple, cu jutoral ae Tei bonpe, Daxia de forma nui cub cave are'muchia de london mai snare dec «penal? 17) Din 12 be de sominge de bembsc sai objinut 2.7 ge, ue Din ete Iilogeami de serine ee pot obfine 8 kg dele? 48) 0 Ducat do metal en volutal de 2 ex? ave a Solum are-o Buen, din 'oolagitolnl-eu masa de A wri ou tues courest intone multe snl ‘Youn considera, sum, probleme in {int de ot doo iumere, tee unsle din ele extn tae direct far ttre aksle'o proporfionatateinvers Fseapla.'O-echipa de 12 melt sgiieah nerd cite 8 ore Ue 1s monitor ugrcgHorind elt § oe po a? Enungul probe! H-sriem sa forma, Uroatoarel cher 45 ‘ i ol desis are echipa de 13 muncitr arte iad cite B ore pen, nhac avem Uemdtoures, Weta. 5 Acum ine multimile 2, 15] si (5, exit proporfonaitate in Frases tetess 125 Wi log Epa opera rot duck akatca amasard esto parten de de on miuneltor gra ateo 2 Decl vem of prublen rin regula dere sop ged Inereans ete one pet potem vfs tn ete le ations echipa ‘de monelter agial as aceagl ogor dace Increase 4 re pe 2, Heelad wtaitosrea schema Tole monde. 4 5 1 3) xia» proportional Atgarore 8 tsi te bathing Crom ana fie paren de gor arith de lfopgs Shiph de 15 poeta Anteg orb Deeb avon o peoblnis te pe Pole pio rele Simplé Ca mato propor obi Be =, donde == Aoeeagl problemi 2 rexolva prin da relucerii ta unitate fay ogo in 212° aie: Doo o eo de 5 meitn gro, rind cite 4 ore pe ai, rt avlagh agar in 2228 sie, adi 8 ale, Ao Fafionament se ayund sub forma urmtourel sehen 45. deci 2 = 512-8 8, Dei 15 muncitari apical, cra cle 4 om ev, arn’ ae on ogor po cate ard in 5 al ei do 12 mu {Ho serial care hueresah Ste 8 ore pe 1) 4 muncitri pot ermipa » horse in 2 sie ern a nore Inerarg cite ore po ai in elt lo yor teri sean frat 2 suntan fare yor lus 8 ore pe ai? 2) Pentre a strnge 000 | ap, & robinete tro 9 eure 2 oe. Due prt a string 1 200 bpa, Brbineta it imp trebuie 8 erga (Precupunem e4 robietle au acelag det ” 2) Pentns «are » sprataya he, 2 ipelones Jurpbe eer nl in a ae le teetout a ne 4) Diy 1 bg bumhao « Inca de poe Jang de de (Sn Ci nto fe pase pot tse din ge bub 5) Ponte ein 240 met pn, 2 tesitare ma ero & lf Ba biliestenie vor pene 19) tte as 6) Ventra a transports. Cary pe dilanfa de 20 km san pl Shel Dor petou a tousporta 9 € wala we dstanya ei ki 7) 10 ex rin 0 cantite de 100 tone de marta rare By Orechips forma in 10 tounitert, pote trina e) hints fn sk parte ah itera! In ne ty se lortina. rae lt Dar atonei ave gi gral dh mip, O eanttate Je? 0, ke zakie trbuic wh fe repr zeta la Wel canting tn parti inet proparionse ew uumirul bo: ‘ator Gecari cantine. Cte iilgrame de ahr va prim’ foeore ea tind daca prima cantina are 08 abonas,« dous 250, ar a trela S008 ‘i notim ou 2, y, cattle do sah co vor neve fini ean tine, Avo zy 00 yi = HF 3, eat 22 580-9= 1060, 9 2 Mai per proved gi este serie: Obnem fa continaare: e+ 15-4 BOB B= 1100 & Psiok = 330 Dei 9a = 160-9 = 1050 0: 31a. 2 canting au aerlagf mama de abona, de exen- plu 90, dara toa S00, atune eantitaten e910 hg ero reper! im diet propor(ousi ct G00, care este nudrl tal al abenator ‘in primele dv eantne, i BO, Noted cw, eantitatie de aaa on yor ren fain tne, ave 22-+2— 3900 ees 300 5 Decl gD. 3-000, 4 = 0: Ts. [Lifes pune problema ort a fi nomena in porto care an saute apart ts par direct proporiouae 2, burps uxt ca ys ne Find date teei amore poitive ob « diferite don cite dogs sun aumir N posit, so tere su se goased tel numero postive Sy, 2 atlel Jag Ne yh tr ire mine (2, 93h (6) ab existe o proporioaltats iver Avo dee gal de po dso epule to gia de rapoarte gale Problema se reduce, dec I tuplstren Aomiralni Wty pli iret proportions ew inverse mmtersir, Bye "Exempla. (png contin 12 bomboane rebie wk fe mpai= tig by Wet cop in pans vers proportionie cw tata. feeb, Pitot coptare nial doen 9 anf ten a ‘Saati ev 29 2 partie ce rein Goch cop vem Dewi La fol se pune prohlern orcte ar fi mumencks in report eu care un aumtr se impart in pr invere propertionse cu atstenuee, 41) Suma tre momere este 480, Ele sat dirot propor ilo 29, 4S safle miner 2) Suma a poten numer este 22, tind o& el lat dint proporto hale tu meres OA, OB: 0; Osho afte mene 8) Suma a trl momors ate 1. tind eG ats ate ein direct paoportonale 4) Patra_nomery naturale siat dire proportionate ox avmeree {2 Shind ll mat move de ob ete 130, se alle 8) Media witmetia te nome esto egls cu 16, 84. afl me lado sia sat diet proporiotle on ese $55.7 6) Sama x ptry numere ete ele 20, Ee sat dee. prope TESS AEH OY CaS Pa Seno aU CTE 2) Trot ume ature sin direst proortonale cu nnmerela 2: 47 es gisteAsee cia ist et dures er 6) Summa 4 rel emere eae 90. Sind re ht iners propor ale en numerele 2: 2; 4 sh se alle mumerel i Sindee in ivers poparyo nade ex inane 0,4; 02, 6840 alle numer, Anerare pentru verttinsea insur unoe eanostnje de hata BY Bie de rere cost 40 ni, Cit cor 10 kg de BM sior or exerts o br Tor ent re de aces al 4 6. Tn it timp pot execu ot exeeuta SSS TEN eRe ro wa na e100 em din reatitate empha poe fia ht em din dram eoneypnde late, Faint dso ow ull c& acoostd comer are Iungimea dem i lijimea'de dan.” wet eoveypunde i '3000000 ont de soca fin teron, Sh premsponem et Int dona_puaete tin tare ate “a tanta ve 00 km. Ce deta a covapunue pe har dw sn o,1002 a. Si revolvim vemAtonren prgblemt Distanya ttre don p BS 8 mmm, Care este distonja diate pometalecorosnastoar dia tern? “cave vance de pe obaetd en scare Pu te unde y = 16000 000 (ne emits We dstanta de 8 min titre dow nets ce pe art cores Inu distaata de 1 ka inte evle dove ponte coretpunaitoue dia ore, 1) 0 distant de. 60 1 se repent pon pan prints segment do lem: Sh se afle scar aceat pla 2) Dintenla tate. dou orate esle repretnteld peo Lert cn sora sono peatesn segitent de Bem, Si se ate diana dates ‘age Cin 8) Pe om plan Intoemit bn xara 14000 un ot de pint este rpgemen- it pratera patat cura de 16 em SA oe alle wea oll do pimint ‘ox Kington ded dss, seat consume i de vansea, Cita vopse Ee ee eee eee ere perme Fae es gil a uaa ive coe aa ae eo reise durin seve’ nition thoes io Pica cat oak uncenee us mais pu eotace » sand ci ariile celor dowd trinnghinri sint epale, si se alle ra- Bea Suet ratine out yee aera 5) Dinas tte dou aa 4 $B ee de 120m Din eet cee © 6 mouatilccl pot termine o Iusrare tn 12 ie. Dap 6 il do Jusenechipa de muneitri se miveste cu 2. In cit tmp ee $a fexerta (ath Tneraren? 1) 8 trdtonte ark 4 dint-un tt 16 rl, Dap are me itu (or 8 trctoire aa trectoars gi le oral (ob Jota Gu ot fuel mie inlrvaial do tsp In care a foot teninntlIneraréa tote confi Tata de silalin in cre tar Hira nimal eu 8 {ectonre? Dur dart aloe de 6 tractoae var fr aiturat nomad 4 89) Un mneitor in ltl fare fac a inten amit ater de tmp 20 pls, ng interval doting 10 phe, [not tp ale muster tt titan pote Site fae piel mane § mn exon im, me, par ey 12 a) Tnvete ile exeeuts 2-muneitrt um numar de pies do dons By fn ete le exeeuts 8 muncitoni 1m aumie depose de 2 ok 6} Incite rite exeeuté 2 muneitor! un numér de pve de dows PROCESTE Sa eonveit ss olosch pent anata. Cin Ja sutt sau gun procent®. Analog, sa convenit 32h toseamnd 2 sie citepte 2 1a ent sam .3 procente, 7% tnsoamaa = 1% Imseamnn 2 (EQ. gi p> 0) 25% tosramna 2 209% tnseurnas 50% Inscamns “2. vem £002, scamne 2°. vem Avem 22. = In figura 9 ta semana un deeptangh frmat din 390 de pitsitele (on loturi de aceeagt Inogine). ‘Cemirii eu atonbe fire 2 heat 18 de pein Report dite maar pat « akin. La 200 ig seta se ste aot de sare th ap lelorhagurato yi mumaeud staror pateifeetor extn inure Pitt gira 4, agora Jew 40 eas sem 4 sae Ms pt pn ot 4, aio fn 400 de sai, te sre = in 100 go stl: 55-100 «=A g} Yom caleula 4 ‘ia 1250 % dinke-o wantitate nsenmnd <1 din abe ennttate. In generat P% dint-o canttaveinsamnnt 2 din acca canitate, Un raport de forma <2 (p € Q..si p>) mumeyto, pot revs ‘Am viaut chp, tse Sit Ave: 1 transformam pe-2 sntran rape de uve p Procentele gi ealculele ou ponents so forte mare Insemnitate eam oeupat den putin ds aownstsehestime mai inant Nom mafia un exemple Frond Un fat tn prima ip hese ealzat fe far. 1. Totus aim 25% in $00 pie atica 2 dia 0 pl vom: 2 900 piowe = 206 pie. AL Probleme so msi poste repo si tn modal urmstor: Producia rezntt Into nt ste do) pire by Ain prota eetzata fate Bunt ested 409 or mal mich eat 900 pes, a 2% in produce realzats Suto Ini este de 25 ori mai mare euit A din produetin realists nto hung gi este decd egal cw pies, adios 22 pie, adie 296 pie ‘Putem sie mai pe sur stl 2. 900 pin = 2:88 as, HL. Problema so mai ponte reas. folosind re Din 400 piesa Ionita pin spt Bin O00 pee, ete ese serrate in pita sop Stunenst bet ad yeeavenees oe 10 piss 26 tre simp Calbia 2 1m gental, deed arom un svonie pivrom ch celeulim 2% din proces sata 41) Si ee eooulee 20% din me de 420 be ) Din Ode eleva we ane 0%, nt ai Cie ote nt Sa ean? Problems. Into cooprativa agricola cle produsti Umble fio arate contorm play intrun anit ntervol de ty 120 ho ‘oot plemul Toe depagt eu 107, ete hetare a fost aut? oars, Ain Soe arste: cs 4.200 a+ 29-4290 ha = 200 be +120 ha'= Atel: Pute si Ccateutam in 4200 vem 1 200 ha = 4220 fm rerolval mai aus 0 problema tn cov tit In outa" “hnalog, ae rezolv o problems cre intervie 0 sehdere eu att ta se tora onsersatit Ae Avem de exapha: 500 = 4 tn intron mamas dat este mera, dnc 4, otunet 2 date 500 = 500, tn gener, aves ‘un momir egal ew numirol dat 3, De sromenes, avem: + 500 = 560, 420% din 2 inscarna t tn gonorsl, dock <2 > 1 atone

sindat 40% din canttaton de marta in a down saptamind $04 ‘in ret gin a (ola spamming ext a 4) Cit svt din tooth marta soa vndt fn a doun spin? din core en iain ‘ele B00 ede int in nbs engl 44 To tala altura ent strat productia de ofl jr nog fant 1838 125, 1987, tn mi ome Awl | 1968 Proditia i Veil meets Hee 1) De cite or ate mel ena prcuctin anu 1087 fas de eo mula HES. Dur fat de eee amu 1963 )CaL La eutk ain prodein anu 1987 sole producti anal 4) Ga lt fy sut ete mai mare productia anvlsi 4957 ata da tes enolat Aube Lneeare petra veritiesea fase anor ennostinie daz NUMERE INTINGG 1. XUMERE Pyrnect weupevey pp Yeweae Tvtmbr 2 Jn manna de maton fel sm Franetul Ol ferent un ‘hferit de panetal 0, netiL dm Tost mai na reget a ncepind de la punetal 0, ou un segment dat. ALY, no en an tisura, ai to sone posi a Oropal in ses ogi al visa drt, einen a festalet Ag alm pe dept ext Jui muirotIneepind ce Is parca 0, in ssi poy al deplt ) ie hngime ete de Uc on hingionen vegetal 0% pal extra derive oO, Incopind de. hp Tagline este de’ dina ori hingimes” sequent 3M Fioearyinumar nate, dfrit ale kam saat devi ye dap 4 (d), xn numdr nota ou =o, prin puneren siniolal 2 face) 1 atoms tral. Newel itil, diferte de punetal O,' sesment 4a punetal 0, jn sens neat adept ( ‘urmitoatte. Este do neces langine cu egmontl msurt, cep at de Je punetal 0, dar Ia sonsl pout al drepte (ay alll oe lid de ti fa dropiolextremitsi sele, dierita de punetot 0, 98 6 fost pt mae Marat satel. Numerele aoiate cu —a, onde o ests un numir natural dite Ae nero, a fst sunite numore Invi nepetie. Nemorle atari te ery fn it ne Tin pes. Nomi Sataral Oe rit feat ic! poeta ast neg nmerslor ntl est dee urmdtotea ule 2, 1,0, 12, aro a fost nota eu Ze Amn pus in evden gi apt cb tice mmr natal estan nie inte adieg Nx unde X ene elie Imerelor naturale N= 0) 4 2 3p 4, x). Uncori mineral Integt [ative eo svi en seminal tn Ta, ede wxemp, 3M foe de 3 Cu ojutorl nowerlr fntregi vom gis elon mullite namie relor Intros ale nor eena{ cam ests eemafia ae eats ili Tn mulfines nmeedor nature’ Annste’ vai youl ‘Soph delinioon operator cw nnmera tte sree 1) Reyrezntati pe a dreapth puncte coespunzitoare mmerdlor: (Se va'hua en vaitale de masa conttoeten 3) (Weal Bg. IV. 2, Teruo inulin tigyra 2,0 tdi tem: pentium de iP: Ce tempera font indies termomstea dl gure, 2, 02 Dar vl in figura penta # esprit cd. wamaral Ting ete egal ow mul Jn genera: Bou numar nig agi Det ogte dacs mere ata egale ‘1a dacd int neate eu “a gl ne ane mt sid ume nate Eglin intr nameel rg a gh Bg serie se citete oe ate elo 1H. Namie intseg « © nomeste primal Py mor fl aay ar moar sa ar ‘ete monbral a dia al gat entra a exprima c& mutsral itong —4 om ate egal ex namin Wrest atl cpu cd mama tseg sate df de mmr Dac Jovi numere ing a giB ne stele, se erie aes foe teste a meet eget 1 a a ifr de Egaitata inte nomee Ingles rai tee mime inte 1 emai pp Acar ete propristton de refers « gels tate mumee rol Ins, in ear se een Tapa et abit emi wel Santa! poste figure sodas! uma ney. 2) Orsre a fbr 2 fi be del 0 = alan ba Acca este propristatea do simeie @ eat att omene sot Oriare ar fi mumerle ine a, Bt dacd am bib m= tuned Asta este propriate do rnc if tre numero Deoetecerulaia de egltat Iw numers tates ard proritila do rfeiiaia snare vonclvtne ne spunea, ake ae tous tie notre mires ete rts lula 2, YALOAMEA ABSOLUTE 4 UAUL NUMAN TSIAPG (ODED) Orice nmr iatreg negstiv este nolat prin a, unde a eat un nundr natural diet de sero. Aces numa, dioph domo it rom ‘un vvlaren abs eau mata names nteegnegeliy ani Intes poativ sa 0 (eto) eta uni nstorl Veloaree ebsoeté sua maddlad ni mis Tates « yor neta rie Basmpl Toa, st a mofaal vnok omar Inte nigatiy seal. i Named —( deoniece |—5| <1 Della wart abuts vat v modifi’ unui mmr Idee pot ents we axa nnatelo. prin eget, ere pornsse di dept noms BO ah ah vette tn deptal ourvtrois & popesty al vero BBs sapeaie i atcloyt fel cu atgwnta care indicd aomiral 2, da este un mma fre posit san tra [ea(dneh a eto on rnd itey angati agorats incepta din vif si naval 2, ane viral 2] 203, cept cist nt ‘nem 1 ath tt aioe BE ta pan ediperen teeter cet Zt oS, et ot Fr vi 3, (oie eve ceo ciel plus minus te, wale —T gr sce ated al) 8 (5) 28" ave isle 4 figura 4 este aatat ov se obfine some oomoreor Antes Tayo sigutade alist Tl ex sigeata care sic wulnivl 3 aoe 28 tala lature 8) In etl semasi nacnit saw -| in uted sl i adv > ex unl et ne fin Ancepind in vio sige care indica | 4) Si we ate vatoeres a aril ruimiral ning «ee suas Humerlo ites ib, we se Wexie & forarin din ae | oy |-7 ayia Bales sian ectrask aac tev a i Breit ni nor luca Slo 0 tea eC ccna EINE ator | Fiind date doud mumere intregi, de exempla 2 gi —3, vom in- ‘uerelor intregi a yi }, in fiecure eax care se poote presenta. | Exemplal 1. Avem 2 tral figure 5 O age ara 3 aeoat incopin fe lead In deep Exemplal 2. Aven 4 tral tigre 6 Saget pet. De toon, ert pu Ital fie sar Exeuplal 2, Aver 0 trata tat In figure 8.5 Ya pone. De acon, Hg ie fatal re ‘cine noreeelor a enh mded=|3 Exempla. Aven figure 7 eat ete no o ‘Carl 2, Namerle intro ag bent 9 erat DE 3 Ts, Obfinees som some ole ucratt a azul 8, Coul din sumerse ttre af B ete mann ‘Suma a dou numerettreg agi b, din care wre ete numa tira, Fired xe muni tire negli ete namaral fires cear se aie ta: Dai 61, atanci e— 0. Dacd [a4 |b fie ¢ difera eine modi? mat nares raed maine. Avec dda pnral or mod’ mat mae tte nomr naira Avon cd decd Fumaral er mada mat mate se na tego 4-3.=5, Obfinenen ont sume este lag i, de acsag ole aspen eure Sadia fin viral sign vane indies caaaeal uli 5, care wnt, doi, sina nuoetlo Bxemplel 1s Fie 5-4 Avem | 3 | = 5 ah Numi latira beste mai mare dev naniral natural 9 Deodreee 5 ante 2. iar —9 esto momar iteeg negate, avem 3-492. Obie 05, Obfinerou acstal sume este ihe & care tics nual O eet reds lt ct at In vl leit ca indie me Exempla 2. Fi vem [8 B[—6 [oie fe 15, Decl 6 +50. Obfinervn acesiet tomo nis adGath bn figure, 4-4, Obfineren abeetal samme esto ns iat cave fnicn numa et Yodae ta cate indi nud 4, seta Ieepl ruil 0 are vi npn, cre eta Fstaptal 3: Fie “5 4 8 Aven Bla | Bsemplal 4. Fie 34 (5). Aven 5) =5 i 13he <1 “5 Deoarvee 5 03 wate 2 ar —Sente mua Ines negate, ava 3 2 Obfinerea aoetel sume ete local fat med Exempfol 5. Fle 5 Astin 5 1 Ooi 5415 tontl cine fe HOE fo figura (2 Reempinl Fie 8 +(— 5), Avem 18) =8 ‘di 8) bl, Deoerege 8 3 est; ar 9 cole noma waturs, ved ‘ofieren acest sume ost stata fire 3 flares i een ue nde ogee opie in ra inter icy operaia de ad dec a = 6 alan COricare ar fi namerele ntogi ay b si ANfel sym, acd ado aosags some inter eu tunel tro tare el dol mom at nei ng re nuove Tate Oricare wef mame iri ay 94 dy dad — 6 ye = date atenbed i Interne fret cbfinem o egablate lates » ss en memnbrs a dou, egal stnert a) ek VT E35 6 40: aN AT ano Bat oym dai 1 Basins 6. oO Pi , 5 oa 0 2) Si ae efoto ne i) 19 + k) 27 + (—13); " 8) Sa se efectuere 4) $0 yresup 4 temperatura ora eu 3 fata de | dovembive cre 7 (Ce temperatira a inregintral po dala do 6 decembrisorcle 72 Fe douk numer nteegs Avem 8472. Ave ees) Deci de Bparakel 0 ited ae poop ameve tnt conde ‘Orioae a (nursed in entry @ pane in evident alta proprietate a toring obeervin yrnatoarele. Orie near ite Aon inne tnt poste fbi ew eran a sep. Poem dase pe—7 4-2 oti —B. Sevin fn pees ene (749 $e5) | fi avo 4 (3) (5) = In, di a S:(3) © Ah, Bieri, hte Aue edt yt Bite erpecat prin rome (742 + ‘tana Hilt eli tnt nrte nt t a (5) ee 744-0) — 5-4 (oy In mod analog, potem serio o soma de forma Freee nie 74 04(-) tar Ds Deri (74.3) eet Hain neta eat ttre numero intra din — wa 0 obtinem, 10 (OO Aveta can tte si —10 en ajgtorl. tans Detain. ‘ita egaitait Inte numeve intr, objincm PEG ch deat the Piel ners " (T4394 (-8) 7484-0 Opus wnt nme ies te nated tay ° © antlel de proprictate este adevirata i pentru alte trot numer Nau eg 0 are to opus eral Er {nuogi,oslato ar fi ele. Accant®propeitate a adunds nomerdog seroaie. Ipoxal vou wan se numorte aseciatiouatea dundrié numerelar nlreg $1 8g eat ‘fica ar fi nomérlelnteog bg cave (e+) emetO+e. of otcegon oR i ean, ern conven prin a atk abe). Decl imtoo de 622,95 8) 6 SMM Velatici de egalitate intee numere intmgis 32 (ae @ a ambit membri af egdbuayi Si considerim axa mumerlr, igure 14 gsi ne fsa ategia tamupra nomereor free 4 4-4. Aveste numeresint egal deplrat : do brigine coordonutelor. Se soune co panctels cara te comepan@ aisha clas thine 1h genet 0 goncrl Deck a Adict,epusul avi sume este suaa opusilor termenilor sama be, altnet a= 5-4 (1, Adick, ditetenta a = b ia SK om alone: 5) 504 pon sa, ) —4 0 opuel at Pane dinesbinrieeias 23 doa eee Hewat whedon diferent i 6 sekoenna 2a d=2+(-H——1 In ogatitaten rin ene defineste uma sue Milena crea din trment il or intergi —2 1 3, putem pine in Tree tint, o slo roar Hh din "Pe “SH mami dete, ir pS ede al “bs eet ute desta 2 Hs ’ ural uematoarle: diferent tntre eres Rents. proprietaton Bache fb alt dod aire , (Orioare at Ye nena ine ave eo at dom tau, aifernta tre 9 by mal ‘Se ict fae ’ Wm lew te ol cu amis Cs Utlaind tiunea de-opus al uh ume daoyg, vou arata com MME Rez proprietaten care rata umnitoorle:diforenta fate 0 9 vind eaten rian ar fC men nea an Yor pune in es Ail spon dnc sod scdasi num lating din mee Dav —2 5, confor definiii sedi uni mumar tte Alinta hon into, and 2 het Bete a Alfa spus, prin setdaron membra ou membru-a dowd eget Analg, 1) Si oootaon Y 8 tO, ay — 17 185 ; (1s 1 (18) bh at — 3 8) 8) — 26 — (24), BT — (2 2) @— (09) a) 68-109, 0-35 1435 — 50 4) pis oun se wade in fg. INS te smomtral din figura ta indi temperatra tie FG, lar termometrl in figura 1 fndisa. temperatura de 2G. Temperatura indies de termoretrol-reprementat i figura a esta mat mane sa tl wied desit temperature iaieta do termome tral representa tu figura 1? Gx ee ) La vn moment dat se inveistens temperature de 3°C. Duy un interval de tiny se inegisteoxt lemperatiea de ie Fompersira some sty «sca Gu elte grade rel tel. Pena sod ste com 8) Si se completers ums | pletat ea mae [Laerarepentr.veriiarea sus ‘nor ennoyinfe de bax Si oe ofetne Pontrs sipltcarn svt, t'Joe do (—2) 2 ee serie —2 42, Te gba cele erica eee ee, Finer en dieengas —5—— Le Dtoith opreti demote Sitate 8 danarflnuinowloe inves in Ine de (3-4 {3 Brn ta He de 34 (9) + (all oe tee 4-8) 4 (ah Ange fe tor de (38) 4 (2) sau tn foo de 2 [9 4-3 90 tr P54 In exemple de nai ipsinte, tnt pvantezle pase in evident ear, aul Hee apc, al. expres rmat dite Tenens din terment desparyti prin smtcurle-¢ a — it in fata garuntge de deschidere nee stool + savin Tata paranter 20 de desehdere nue al net tn sibel Fiera ator reteyiecie female tira 54 nuptind pararioca de dhechidere # parantise de tnhiders cot fn cusnl care tn fat parantest de devchieree aft Simbel ne lace ve sprind dard potnal sil el prilal tonnen expres fete inte jaraniseesimbtul CAB mca sibel se pbs Teta) 1, Prin dasficeret parentersler Uropta, expreda (8 4 = Came (2 devine 8 (0) + (2). dn fafa paratone dept dhe dotnet se afl intel Exempit 2. Prin desiocsen_paraatoaeloe drepte, expres. 8 = ‘oo 2}} de eL (3} Am anprnat sill dia {ath paenoter drepte de doseider, deeares peal smbol al ter iment by care ete prin preset aflateintae pata tenn rept este im empl 3, Prin dsfacre Cee leche re ifs rotande de ‘deschire, deqatee pital sibel ll tryna by eare eto prinwl ternal edpesi aflate Tate pranoaele ro finde, taste stabil —- Si consderim, acum, cao suprimiritunei peeehi de paranteno tuire cares fla expreie format din ferment despa pea ors sai, kn fa pvontens de descidere 4 Ai si Bolt : Ta Joe de — (0) scion 9, Into de 4 )serian = 6 — arin oo de (83) serie 3 +3. Am jl sean de to Ani in ‘care mexprims opeenl mui tum tnteg, opasnl me tous nomere ote! gi pal diferente a dows numer iatret {nr esomplee de mains, Int porantezle use tn een Paste bran appre lt teryrete esol Sata en ena fin ferent despre prin sihalrie sau — a Tafa farantevel do deoctidere’ walt skola tn acest ean, desfceen porantzar const tn urmitoar Te MRLATILE: <<, 5, » INURE NCMERE TYRESE Se supriod peranesa de deschiere si prantesn de inchidere Fe eerieet dat terbents se noevese ca sibel —. or fete Fin o perch de numere intregi, do exemphs, —2 gi 3 Deane = alse ini ene sa : f vehici ee karte at ern ne ese fold ft part Baad primo omen (ee ibaa MMM oe 5 ete wo node intrg posi, se pune ea este al mare dete snfriminiie 2 act sibel one tld i os vert ecu el dan fagepranteri. de dexchidre 3p Fremplal 1. Prin destceron parantensr dept, expresia 3) Primal stolen : nto In gonera: In loc det > 2 ne mai serie Init, ea ete primal termes al exp ? Arpt fil imal, 0 men Umum ites a este mai mare ditt wn mama Sebald fa parateae droplet ie sre feral de iba se pune smb en ae a>), sep 2, Prin tel sitezelor otal. —(—5 cee e Pa Primal simbel a tema 3s tart gate. ek oe exis wn mama tnteg poste axe ine fork w= be Sinclar sevnnesi Fao vine a expose cae Flay en nypen Tita parantei rotunie Bede un oe tories ew smbol pent ot acest i ie det eur de nepalitte De exermplu, “2 ene mat mc dech © i ve consala po Tignes, feu 2 ene ia ange a 5 . ee i bo 8 2) 5a av desta toute paratezse, Sb elecueae eau Aven <2< 1, —14¢4,0< yar 1 = 4. Duet notim eu a oiewe oF eat errs elo intregi —2,'—f5 01 avern @-= 1 sea 1. In oo Batre a1 nana ~ 1" vont vevie a=, coon cea tilegte ye ete mal 5 — 6100) it etal cw as tn general, find date dout meme ttegt 96 b petra a ined eat eed oan Brien 4 om 7 tal tin zl ‘ot inate nest ajutorul iiere Autres este Urata: empl. 1 Alsi cdullaé unui nama tate a, duck «03 oy aed <0 rena 1) Reprezentati pe o dreapia momenle: —43 0; 4; 254 (rant ca unitate de misura tem.) 2) Se considert 1 numer intro cons cle rata 3 Care tnt elle nue 4) Se consent 7 numere intrest consecutive, Numai 4 dinte ole 4) Tolocuf in esle eo snmeaah remnel ? ow nn din sole et Gate neh cbtie(t proporit dowarate 8) 420; by —42G;, 6) — Ara dy — 201; OF — 200 1? 24a; ny r10 | 20 ve. Cal mai mare dite fa 20; 2 Tai io; Diolzo; by [oita; yi —9}r—9 misit)— 5) Si os arto ef, pentru oreo a & Z oven: NTO) tea ela te tate. 6) Si se efwctuen Wb (A; 2 9 trl re Zyl x] =O) Richaniieew ied 4) Sts repreziate fees dintre urmitairele mimi, enumerind Semeatab al yA {s\22% b21< Rariien ee fs|= al jesiseem as—o |s)o—a) Ddoeleen a> 3 “sza ey . boweurines Tetalit tn numa Integr otal ou 433 ga 3. Acosta ge Rina {ined sear ck nbeel inrmgh& 13 sin numere naturale tel thd produsst Io, st cut fost Wein el in manual de oat rate” pnt elas Va. Deck 3a 64446 —t2 Spmem of rune troy 12 + ebtiautpin tomo numerelor inirgs 3-3 ane he itgt patra on toe ert EL saccata asta Namerele Inte find ponitive sn negation sau egobe em xe, vou da lwo dni penta bijineren prevail numerclor intregh arg in Tecnu cee ponte przont Caml 1. Namercl itrogh af B sot qumere natural, Prous a dowd nomen iteg.a 3th care int numere natura, rods rumeretor ature @ xeanpe ‘Proiural a dowd: swore integi negation a 4b cold Seaieol destrece {4 sai 4-312 Canal 2, Unal din ame Prodanal eden numer Tore a gi din care want ets nude Introg gi A este namie natural In cas contrar ¢ = — | rapa 2. vem septal 3 Deeper) Exempla 4, Avon eoaroep Hollen Taek’ ble sar Tyebuit sonal Tezuka propeetaton core arith iy. produsul otichui mums fntreg ou {esta opusal scelat mimi ito, adie Orioare ar fi mumérad inves @ arom 0: (1) = (“tyra In lor de aflaren sou obtineron uni prodos de marere tteegt ‘vom mai spune vjesraron produsihi de mere ateeg ‘orn pune in evidenta mato doua propviet vin rltin de ealitate nce numer Tale pers B eeraer foto ‘risre er ft numero ite a bie, dad fo de aralf Aldo spas, dacd'nomerle inte, ente nt cl dol meme ai neh epi tatre numer Inte le Mimulfim eel Intregobfinem tuners intregt es rear fi numer Nfl spas, prin lines membra en galas ‘bjigem o egaitate intee muatee nteg Sa 20 efvetuen 2) 4-8); 3) 8 (95 4) 3. (— a 5)0-9; 8-4 0: (3). 8) 845 1-8. 10) CBs I) 1 (=; 1 9)-(“y yy aye ht ANMUUPIETATA De AMESAI SI SC\DERE. REIT SECERL er tog 97. Aug (8) 7 8. dea © tte de proprstntn este devas ovate art preton de mers tire vametlerne, eens prapretaien inne mci Intel se-numoyte comntatctatea tame numero te Pentru a pune in ovens o lt proprietate a Snmuliil nnmprdon Intel observa urate: Once mumae lala ohn x peo 4 dou amore intreg poate M uUlant et factor al ust produs de ‘i namere nt een toe Ene adevSeats q@propristatea care ve aumojte diriutvttea Anmalji forte sehr are to cons. ‘Orcare ar ft numer inegt a, bf © oem ‘tei (7). om float trataltin TIntradevis, ob 6 auimets akg, detoria dats iti EBSICIS ines Rae ages = nde di tne Paha a ae ma i " care este un numar Intro ‘0 obtine astfel . ee ele oe F Decl ((=7)- 21: (8) = 70 #4 {iin egaiatit ttre. romere iat Cam ote em WED ( 3) Apo din (7): 2-5) 90 gD ‘or ajotoral tranetvtai egalta fate numere a ba~ fata bine on te ale tel namere nmi nomereloe umerolor Intogt 9 Observe 0 Bite $55 EM La (9, 1-4-3) = (~0h Aesestdqropicate expr fatal oh namdral ttre 1 ete element Din aceasta cauzi, vom convent ea prin a. intelegem ort ee 2 eee Nees am 1 sven Deiat (CAM ac (-2-(-10) © fel de peopitty ote evita poten lo, te namie Terre eat verfenes ie) ener emf de had eee ls edu. 1 hinted ite) ci dices de arcaeg shectaa begat: Etep Suny Grate Sree (os de MMM AE) 2). (=-5H'2) 2-1 o, fare pecans eo ices ae linent Cag ore. Os (st eo Co eagta 2) F(a) -ea) 2 (aN WS Ewa 10, Dupe mn egtitaton prin care defiogtaprodusl msmerslortnteegi ~ 2 957, pu tem pong revidenfa eicore din factor arten 14) 2 yt) =7 Spnom of —2 eae ital Inte —14 317 obfinutprin,impartien al iia 7. Pe ta Hn dim, ar et bupdettor, Woalog Teste tial opi damp Aa" impetne ae fal la care a fasean ‘wt nimie. tres, dar nowt nul siigu intoyi, do exemphy, Ll, wien th gene Doct a sib sintdoud namere tres afl Inte #0, ta tne agi dy moat prin a: beste vel nan tary eae ext, ‘Vor spain Nerul propretaten rare arth nemtoarole; ital Inte oiee nome Itreg pi eae noel momar Tate. adied ae Avom Renal proprelaten ent seat’ rman Intter a! (if este opurl cel amie tne a(t) =e Aven | etaten care ruth rmitoaree: etl tatre 0 4 ote edt de ser, este 0 ads, imaral HOE ws ier a sre, quem Feat prop orcas * Vm. pane in evident ae : relate a atte nie numere tee op ‘Orienre er fi numerele intr @, b xi ¢ asf ineit w— b, 6 # si Diaries fe eau tree fl e-pe deporte pitas flespe re at oe ee Ail ops, prin marie ou nn aude nto 9 aabilor mown ai oul egal! Inte mutes inteeg tunel elad tmplrinle mp fetus, Sojinew 0 gallate lal aimee. ited ‘ricae nr fa vamerle integiay b eg. ate ot © — 8, ¢ —d ev .d'e 0g knpltine we ot fot tne axe pede porte aie 8 ged pede ald pani tunel aie 8 tel spin, prin iplrfieaeneniew ou mombru. 4 doula lui nie atmo atregi atane lod imple ae" po efaca tbjinen "0 egltate Inte ture sae 1) feta ay Get By nat ty arrare pentru verifiearen insist unor canortinge de bast | (ap (0 226) 1 as) Aa) 0-8 Ty) 0 1) Bfeotusi | Resoleare. i 8) Btetust: 1, prvmonmt rye New REG Te numetels ntryi —24 gi 7. Exita nomad introg —2 asthe aoe " Pile mineral eg oh peste 11-2 ta cas ner Bt Sockets Into 45, ext mama eg sti nc and a erence niet eek tee ame Resalvar, Rea Ee on etoccmyee ‘Se socio b | « gi se mai citeste ,b divide pe a* sau ,a se divide eu *. ae tra eft nu existd nici un numér intreg astfel ineit: inmultindu-] ew Bi 2 xi obtinem —7. Seriem —2 —7 9 citi ,—2 nu divide pe hy 0 i 2 xt obt 2 2 nu divide p Divizositi 6 dats 4, 1, 2 —2, ~3, 6, —6. Divizorl ot — ut hy 1, 5,0. Divot Wi ait Ty 4,7, 7 moe ‘acd a ete onde natural, nina 20 ete un multiphy do a. ae De cormenes, eee un mally de 2 Mai gpanem ef numaeal 2q In memnbol a eile a gabe ce ao. en mel 3 i i act eg ot numer ttre’ apanem, de aremenes,e& num Looe ot e+ este divisibil cu 2 Inte-adevir, putem serie 2a + 2i gene 208 madam (27-7, 9 prod a+b), ie (e+ 0) esto un mame divinil ev 2 Sr 2 cer it ipod Raa ce neces de eld Var te 7-4, I mesbeat oe al” ego intr impar este de forma 2h +1 seu 2k —1 (F €7) 9-8 tM) = 2) = (9-0, hice bur ng tte do din oe agg pags nt le sch et 4, 442 bom, a Intreg este de una din formele: 3k —1, fiver wade be |" Urmivind tsbelal alatarat, vodem fuani dale 8 i mel inkl det Nameerh —7, =, —1y 2 sat de forma k= 1 (E22) Sumerele —, —3 03 snk de forma 2k (k ©, Numero —5, —2,4, 4 sat do forma 3k--4 Uk € 2) i107 2) Slnd cd ab 0c 89h ead 4 sso ealenloe 1%, INPARYINEA DE PUTERK EC AeEEAS! BAZ 14, NMULFIMEG Dr PUTERI ev ACERAgt Rizk tien (2 (2 i pe (2) te To wn uma seg ante] Suet dae} Ini ‘enna ‘deoaroee Dag ete wn mdr tnt a Jn genera: ‘Dav a este an nur inte iferit de, or min sind mamere nate Dect Produ puteior eu aevea: hark este 6 puere 9 acleash bane far exponent este suma expouenfioetattrler pie afl tncit me > alae a 3a mw (tut putvitor en aceasta eterta de 0 (exponental dempare Malu tind el putin ec e% esponentul paeiora, ee 0 fatere a aceleoy at ar expel este dilerena inte exponen Tu depuis exponeatalInpargorall U—9VF = (97 (—ap (a ser . Jo genera: Dacia ete xn namie tr ar m Sh se efector: a) P45 b) nae 3 tape rh) D oem mas &e2-2 ay yey pm 2 1) cae ae — wpe Dest terea nul numa fates se raid ta © putere pst bazn_ pus {eval muna iste aint exponen ed tr erate pontew verficarea insti unor eunoytinfe do har 2; $h(-aP — 7:6 Boe (apm, 8) (2) 10) (3 (3s a) CPP 2 ey Ty OP, 15) + ape (0 | Pateo sti (9:2 = 1-2-9) (1-2) ps l-3P 1) oF Avominator, {Un predas de momen Intec seri tao patere ridciad factor Ia'2eeputere st taming pater dine © 10-2429) 9H +2. ( et 1) Si te ealolee ID) ot etme 9) Sie cael: 10) Si oe efeetucw re dia wrmitontele muilin ssid elemem 0) Si oe fectaere: 12-31) 4-0; 6)(—2)-( Hae(—as gy 8 4-0: -5}-0: ni n) 10 25h) 4 ie woah no 4m) a) [mal 151+] 0) —3e 4 Doinseh 10! a 200; Ea(sleen, #10 Ge laments ‘are firare dintre acta thllini? 8) Si pe eaeulo: (289 5-28 — ay 8) Xotam ew P predusl a cinel numereintogi eanseutive gon nj Daod PO. ete valor distinete poate lun S? Care este ea tal mich scare ent con fish mare dine axate van! B'baei 80, lta valor dstnete poate han 10) Fie 4 o multime formata din 20 de. numerefatrogi conseutiva $059 o multe formatauin test tere ines onset Enfant mir ain mugen Bete en mat mae elt tual mare nur din mules In arial db métemalicd pentru elosd a Va, nomerel rato ani nenegative om fost introdave semua sol ear a fo Piisodgae®numtrdle intregl Anne, et considers © dreaptl ( Tigra, yocnroe fort un pins, rai rine ecundonntelar. Second! Mreapth mai fort fxat un pase citert do pumetel 0. e drespta (cent el ponctal Ota nctul x et imi sad refolded (2) or senaul open Tost- um ene geo al Arepteh (ah Vie wegment dat 0A, nit wntle de maserd, am Ses in Ace pUEi do acovesi longline obtain wn apie, ALP rede tats 0 treme cin egmentl MN. Tncopind do Ja punetab 0} im caagrat, i sensel post al dcoptel(dhn segmented ffm rate egal ew de" 3 orl Iaugimes angmentalel MP i am pis numa riionsl in extremitalea diferita de punetal O, a segmen Numai 22 dintre elementcle mull U 2 sint negative: Care # @) fate eel sal mie numa din inen AU? Dar el mal mates te) 11) 84 xe arate o6 numero de forma “8%, ey @ EN, aint Fev mere natura. Noli Pent spvotundar eonoytintelorrenteayi gt exerci gi pwoblmele de Iu 1 la 8 pagina Ie Luli misrst. Jn dooptl termi, diteite de panetl 0, ane Namerale rfionale agate pe drespte (4) in consul posit Je-am mult puamee rationale pout rcupad de" le putt OL am tuisurat, si im sens nati all roped 2) wn eagoont sear Tungne sate 2a 7 OF Tinga sn dreptn pmeatului maura ‘ideo nmr ralsnal pot fam axel, de, po dreapta tim mtr natat com prin poner sibel (tous) in fata ml rationl pity. Namiral or fost pus drop the inilatiy iferte de punctol yn mented murat, ierpind dela cel, Tn el opty a opt (care egret aig towel’ eae de teat ingore segrbental manor, Sp theta pumetal O. dar in senul port al drepel (0 ata fe Aleptl extrait el, dferite de punetal OF 8 fest pow nmr sal ot ee oe au fost mumite munere rojionele nga: Nuxateal 0 este wi el ame Fetional or fins nie ois el negate, Dios mie atonal oui sa ae este nuthit ayn raonal nee ‘utjineanumersr raion «fast notaeh ca Q. Am pus ta evident pi Taptal 08 orice mmr Integ este un sunae rtion alien ZC°Q, undo Z este multimea mumereor Inte! Z = (. 10102 3 fy es Uneor, numers raflnale ponies ental APP, care esto teime din segmental AN, gam pus remit dierita de punetnl 0, seria n srmnal + In fla, cn de exemple ++ in Too de jatora nanurlor rationale vom giro im nulimea nue relorsifionale ale onor cena, cum este wonafia 2 + = 1, care ru au acai tn mullimen nomeror rafionalsnsnogative. Aceasta ‘va fi pout dupa definien pera ew nomere-raouse Evert Repremntati pe o despa moment: Lief on unitate de matsurd docimetr nruaTi be neato: some StH RATIONALE Sera eet ‘Yoon sei tu Daca d = 2) Ore 3) Oricare ar fLmumerleraionle a, fie dak @ = gf Bam on Neeasa agian ntelogem pn aceasta 8, pentru x sjunge dels Ia wu dumae Intex feast, parcurgem nxn hastersie in ns negative woprieates de tanstviate «ego fee nue ‘rico numa raonl poiiv aa egal cu O exe exprimat prin Desarecyreafin de egalitate Sutze gumere raion aro propiell unde lb wut numero noturale, far 0 este diterit de sto. Punind vale do effeioat,simerte i tansiivate, st spune c& roa} MBM emma (anus in fafa fiecdrauusae rational poaltivy am obtain . glia Into numero ralonalo esto 0 rela de ec mules tumereor rationale negative, Pacind feuniusen lin . Humrele rationale negative eu mullinesssmerla ralpala yet ou evale oo ao, we bine mallimea umercor rajonale, cares sr INeLUAIOMME Wen ce utes pein Milkmen namereoe naturale, notaté‘en Nox urmatoured mum 1" 10 Ho am convent ot seroma. Tn come wos, aroma of tine ‘vow numa natural esto un mum rational post sau egal eu zo 0, 1,23, 65, nied analog, in oo de — scare este un nome rafioal negatir, ior mlfimen numevlor Intrgi negetive este mulfinea Fonds «cate un numir naturel fered abo, at couyeit srr . unde rational nega. ‘Orie numa Sntey este dee} wom in 25 ‘Dar existé uamere rationale positive care ny sit numero naturale 2. tn adevir, boy “3-2, Find seuniones maltimis numerelor tutrgi negative toflonly erence we expt omoreor natgeue oa ob{ine moltimea tamer (nz, eane ene deck uomatonnen tli a , 14 0, 45 2,3, 4,5 rice muni natural este deci numtr iteeg, esta ce ee exptiml| Un sitfel de numér rafignal positiv este, do exemply, Sind to Sutre monerle ‘nomieul rajional 2 poate fi considera Beira 7 5.Suck monte rail 2 leon mk nal ‘unc ltl Sots nomereleuaturale 7 9 este a Dect putem serie Fe v2 Sa, Dah <2 tunel 3 «<6, iar daca eS stnei 9-0 =0 gi, in agit, deck @ > atunch 3-a> 9. Deci propuijia cu o vivisht 72g cate fakh pentrt orice @ ©. Dae sic om mom ites agate nu ete nade natural dere Ahemte In dreapte nummaruli nalyral Dar nugherle falreg negative int aqeetve Ik sunga numirulad natural 0, Acsetn tgseamnt a Tnullfmen momereor naturale este o submulfine seit a mln nmol itv ceea co's expr. pri em. Pero nu exith nie wn numa natural care ot fie egal eu 2 [Do wsomenta, nu exist nie un numdr into negativ care aff egal ca 2. deoarece 2 ate poritv, Aceasta ineamnd.e& mulkimen no oerelor intregi este o submuljime strict a mulfimit uumerelor valor (ied pane cum numrnsturl, diferit de aero, ve afl, pe acme =e ** e=vrI prn meet teat numa natal 0, Salegen i aerate 40 ports ajungo deh O lun nuinde natural itt de 0, yarcurpol Pie mace oh aie Era thmelsie tn wns posit, Gad apacin ef va nute tein I1oede NCL Zc gait afd, poraaa wumieotor, hr tings oormarolut natal yeneg, [ei | z ° 2. VALOANEA ARSOLUEA 4 ENUTNUMAR RATIONAL rafional pegativ este nota prin =a; we @ Votoaror bert nav moalel vnvi nuinie raion negetiv a i “| Esiapla. Fe « Avil Definite enor hnlnte nan 9 modalataé int nome eofinsl ote fedatay deol ob Torna dad a eate wn nam raion poy sa ath sort 3) Sw cactus offs -$] ei iat | )|—- | 8) So coms compan Suma a rationale o vom defini i mod seemindtar modo in caro am defini wim « dou duunereSntrest De. alt ator aseei, peace ‘¢-mumerelor fatmeg att perl, rezltatul ade omereor intr conaheratadeeptmureen ional, rom tal adn lor Sate trega reel nimere elutes consider ch huitee int, find dieie co repultata ature | Dec, find date dous aumere rationale, do exemplu 29) 2+ vom tnfelege prin sama numerelor rationale 2 4 — 2, caro co ni age obfinem in modul antat in figura 3. fn aceasté. fig, amo tele 291 = int puse in evidenta, pe axa numerclor, prin st fe pores din dreptel nuanéruui i au vietriefn dreptal my ere rationale postive ate $s duph com se vede din_prisnl Perlirae corel Ud dane 'a Gisnerche. raja Nessa eel ee e gle eae acral fiona arab 2 respectiv al nindeulai —7. 0 sigeatt de acelai fl fi 2+ Bsplicnen gata care indied numérul —2, ayezath feopind dia vel a alts exemple ra face tn mod. asemititoe seit core jaded nome 2, tre dip nana i sal 2, Spann of mio ain primal exerphs 0 obfinut prin all Cazul 3. Un) din nomerele rationale a gi 6 este numie rational atte eat egal ci ano, tr ellall este omar rafianel negate Bee deed sect bel le bei caries edna fe ional posto sew egal cu to, ier ella ete nam Poona meat re tat atin Scare sic Dac e\ = 0, and ¢ = 0, Dac [al [bl fied difereije ier Pandata mai mare’ jf modded mal mie, Aven € od dad andra fer mold mal mat tate Mtr raftnel posi, tem e = a deed raves numerslot rationale 2 yi — Pentru» nu no refer de fecare dats a cit o figura de fal ele rai aint, ataneleind so pone problema saunas dou numery ge ENAMBME 1) Aven rationale @ yi 6. vom formula cite o definijie pentrw obtincrea sutudl 4 +(-1)=0, ‘umerelorsalouale a q bia ficare cate fo poste earn K Caaul 1. Numerle rations w 4B sat necegative deoareee|4]=4, |~t)—tyift/-|-4 Suma a doud numere retinal si b, care snd nenegative ete a" ‘uaa ronal care se vbine ost um’ em arta in wanted dee 3) Sveum ‘matematin petra casa Vea Ha2, Dsemple deoarece [2[— 2. |) i $22, avin a: B> 9-4, i aR ge eats areal Anat Ee vi poi ci £ e mit raional posi Carat 2. Nomercle rationals « gi 6 int negative 3) vom Suma a doud mame rtinala negative beta rounéral atonal ee ee ced fal detieiée [a8] ‘i sind 52> 3.4, iar 4) Anslo mn primed exempla, valonile absolute ale mumedor rationale Iv ie de fren oat obfinerea susi sume de numere rationale ‘on nal spne foataree some de there rion Inti Te pulem comers fon exeuple de. prop Ae natant a rsnetlor raonal nuns it tational. Ne puter rele esate exempl ndunare eu unter intel te few flo lavlton adundrs nameclor rafinale ee enn al oper ionsoenopeatn da ‘ar fe numerle rationale a gi b avem at dnd +a amerclor ional male a, bgt ave a adunit ouheeterg ee Dio Gibb te of eh G40 Ant In loo do (=i stat wat caus, vom convent en prin 4 8- € a Ine ; = sn ede 3, 40 -f40--S 04 Aceastapropretata exprimd faptal Ipeal ned Te encores site. umral ratjonal 0 eae ae Zon we eet ‘Orisare ar fé mumrat rena a wtem Yorn inti pune. in evident wrmatourele dowd propsietad in care Saori nt date it momere Paine opera de t rear fi numer raponae eye dacd a balaneé a+ ¢ = n aclagi mumie ronal cu aumpercle ‘aginale ce snt ee dol memba tel egal intve sumare Pein Tae obinen numer rajngle age, | orf nanareerpionala a,b # dy dab a=) si = Aik spt, pein alana mere ot stat ote nosete tonale oblinen © : nde (ah n(-$) eal ote(-t ++(~4) » apy Ho 5. OPC UNC NEVAR RATIONAL OFESEL UNE SE aio de origins coondonatelor pe ns {ul edtora ge ala norercle rationale avspetie.— > sat siti fa4h de originea coondonatelr: Ds sssmenes, se spmne et — 3 est ub ui 2 iar & ate opusal tn Dente, Opal ani momar roi ional nagoio a ‘Gpusal nei numde retinal neat» —a posihec Namira! atonal are ea opas ra ‘horror. Opt unas wom ralonal Deeidaci a = I. atunci 2 Dred a Deed @ 0, In conn! i care a — oe Duchy ste ond numere rationale, diferent trea ot By ot in ah eats cal name rafonal penira ear eguitaten —a 3 se ma vie (= 4) 3. De — ‘pusel nusarala rational —. ‘Tot atfel, deed @ —0, aun Secantaih 0. Deel 0 esto opus numérului ttre 0, ore sie citogte ye eate egal ou « minus # ‘inn! Hofinen de pwn A wok ume rational, difereni itee ai'nuimere rationale so ectieast asemanitae mosttal in cae Fe fvturartifrena Tne mamere ite, Ateata te datarsta Tap {ni'et numercs nee Sint numere rationale ie operate de ad hve tsetdere cu mimes intro ss fae in el eas aunt led econ ‘Mf te cectuan eu aumere rafal. Ua exemple dt itngel vind vorbit de diferent tre muse ire Tn general ricae arf maoverse ropionate a $B, acem a In conor ‘Sma nie en numds rational i opusul si este gu en 0, ato Un exemplu, de dul in cat se abtne opus uni sume le namere Fiionle, so poaLe da cu o ama dewimereIntragy numer Ines Fi pmote rationale. Un astl de exemphn «dat tunel eins sorb despre ops nal sun nv pte tne rei Falcindieonsiderim ed efetutin eu numer rajlonales In goneral: a—bmet() Aiiet life a —b ite dod mamare retinal «gb este eal Mine E tee nur rationed as opasnd —b ob namdralt bina, ) Si oe dune — 2 en opus si Tn mod analog 6. sekpenes, #28): Aeosrece nomiral ratonal poritiy 4, car este ealoneea absolut a pin care se defines sui suserelor rations —t si Se poten (alee: sonore poate lal ca = este diferenta intre + gf —2. obtinutA prin sedderea ‘ us 4 din. Pe 241 nomim desea, iar pe —+ sodedtr Analog, —1 est ditereata Sat dette 2 gi sctaural tindra tofone ry Dea, dock fatre dou parantens to allio exprosa tent deeptetit prin snl “eau — iar in fl Sedeschdere se afl rola -+ su in fa Sng fit nik un smbol, derfacoea Exempla 3. Prin dosfaceryn parantezalor 4(-ay]+(—4) devine 14-0) 4 (4) San imate eee se peri ds TBremplel 2 Prin dsfacerea poranezelor depts, expisn + son +[-2-+(—2y] seve 8 4(—2): Sn mint mle drepte imyrana cv simbolal wit din mumerele WB Gg derhidne expres. ala Hate 0 ater momers ee WR sehimbat ds decd a= 4 icemplal 2. Pein simboll din flap denpati np sani ea ir infte pranter Ved cbole es ostacercaporanrsor se face Bsempll 1. Prin destacorenprantrshon drepley Fp 1 (6) devin 24 2— (A, Sa mpsinat | f ale depo, Sibolul ear despre ex do termeni = Simbohl — care bn al tertenit mere tog ind numer rina, iar operate do adam, fs, nuprimel Imre me pa cizan cn munee iibesle vom, coos ola farsi mine valastancl thd desfocereaparenteror 20 aan Paes = tnt Intro parontezle eotinde a fost Inlrnit ow sinolul Simo fn generat ie mumar rfinel «ete rai mare det un nina raional by lermen al exprsel alla intee paranerse rota, a fost auptuat Inpretid ou sibel! — din fafa purantese! totum de desebiaaes Exetmplal 3. Prin desfacere a>h, ce at postive axel tnt Simbolol—, carw deaparte el dat torment 3 yi 1 ai expres aftate Intre poranteaele itunes fost nloss pasta simbolul di a fimbatah +. A fos patonterelvotunde de dechiden A parantezlo, Si me elocuean calcul razpind aaa avmerdar In sina poxitiy. De exemple rarer stata pe figura 4 ck 2 eate bn dreapte tes ELATILE <<, > TNTRESTMERE RaqrOATE a Analog, poler apone ey pe axa sumersor, din dovk nine ratio seh rt eta) a eins orate ial ms fT sgh eat mal mace neem, yg ae 2 yl Mikal al mie se foc parsngind aga nwmetelor In 08 De wxrnpin, 2 esto mai mile dett 2 ose constath pe figura Jor 1 esto on numte rational post, se spun ck 2 ote mai more en Sesto tings Ini 2 wt. Deed notim en || Tn too dea sere ye < + gon @ = amc Si av afla valoarea de adevtr a fiectrela din--urmitonate propon In general, find date dond numere rationale a gi b, pentrs a indion Nae 4 faye Sac ses a Patan 2 * Pree el eteaen es 265, a 25) -2>-4y cena oo a8 citeste a este mal mic sam egal cu " son 2 este col mult 5 Analog find date dan mamere ro fapal a= sou a = sem male si By petro indice ae reeling lama mar au ui Mee pin prota nucle rosie 2 so ert ‘ori produsutui, un numa rfional notat saa AL cate no inind arama ed namende rajlonale © si & slot nnmere itive, sof ration in'manl molar ration patra ol elaci pe di Namerele rafinala find. ptt oral 1, Num Pratuanlo dint namere raionle fv monet de mate Exempla (aan! 2, Namerelerefionale a9 lat numere raionale nosative. Produit a dand nimnere rajonale negative 0 yb 0 ete nun ro sional posts e= 1a {18 Prempl, earl 3, Und din sume ° porliy ton so, ae este nimi rational negativ Produ a oud numerereionale 9b din cave wna ete nama rational postin sat or, tar ell ese epi at ‘mamiratrjinal core soles ete Peace Ohig So tyataaet oe tee eect oT Exemplal 1. Aver aronreee|— 2) 3 92 Eseaplol 2, Aven Exempla Avem © (Gal Asoatees primal fortor ete eer. Bsemplal 4. Avera (-3}:0-0 Arograce al dellea faoor este 000. "Trebnie rofimt ca Creare ar [i numa rayonal ave 0m Oana, Se obserea ot: Foie 0 (28) FRemlth proprotaten ente arath. eX produsol eu — 1 este fled nnumisralfounl st opesa! ace nome rfiogly dick Cricare ar FE nuda a 2: (=f) 2 (=t)a = =a Th toe do aflaren sa objnoten uh produs de numere rajonsle vom, mal spune eetaarenprodusuut de ners fatto, Vor pone in ent rmatonvele don propre near inter vin plain de egnltat toe ners rationales oporaa de tnmulie (Ora te [nti Live ALT apes, dak mumerele ati si neh egal inte suanere raion ational, obvinem. Orica or fin amet aren bi ‘stl npos, pein Smoove rember ex sntrg numer affoale, bine og int ost det mem 6, coMPRANVrnATE, asocturTnaen, ptsmammertsTeArT, ple do propietali sle opwafiel do tumalfire = numersiot f atpalem Eomsdert a eh ekernple de propel. ale operate! de tamale a namerlor rationale, nemere! Inte! ind numero roxlonlas Ne pata polert In aceio eamnley decreco oferatn. lemulfze ou alae intrest te Tae la fel ea mune ein 0 coma team ea o fare eu numete rationale Comutotviteee inmaltir pusnereloe rstongle #e enon (Oncaea fl namerele rapionate a $i b avem bebe Anctatietates tnnitirtl womeralan sid ‘rivae ar fo nmeel r ‘Din neeesti enush, vom convent on prin a-- ¢ a nfslegem ori cme ee (edema the Aa ins ae (= 2-7 5) sma (-3):7-(—f) Sedememene, node [ ] vom 1 gunerl Prdbaal dint wn mymde rial gi incense exe opt cf 2 eat ttl intro Spunem of rin tmpatr ba 1 deerintn, aur, inven profum 2-(—#) esto Sais i, er po impli Aly inversion fatortor © yi ai astal prods. SA nota 2st lt iptiitdimpdigatat —2! a tmp —2. Tn (-F are hee mpa nte dond numre ronal nen ek Se diorit ds zor, Stim 8 pret mbit membri_l 8). baad Impietre ee poate nestua inp cele dou numeryrafionale ‘Na putem impart wn nude rational cw 0s Tatendevtr, on sk sin es [ 1 fin b invert a, care Alec tmmlgind, mak ati, cu 2 (0 tivbuie existe un nomi raion a astfel fel, 1-8) obtinem ab =, eae esimetria elit d q 2.) <4, Inmuttind, apoi, eu 2, inversnt tui bine een enn se pont, darvee a: 0 = 0. Nu are sens nic 0:0, deoaree slot mai multe nummer rationale, de e np +3) ante ambit membre nnett (=!) 0 =0, 4.0~02:0=0, {general Dacia gi b sine =f obtinem | ~ peices sau Fete cel numa rational e, pera Am june fie lope ye eta og en a pai a Utilaindnofiunes de ivert wnt nu cag inte doa ner (-3}4>-@ Inver anes prods de sitet Ge san eke protust tavetr fact 5 oyaitay eu nasal 11, Durkin tavern ti 2. Ave 1 epaitatea ey, vind tn vedere ts hs rationale 2. gt 2, potem prin care se defines produso! nimers foes lin factor! In itor pune tn evident oleae din eis acre clr} Gs) Basel fiona a un mar rations, avin ( ae 1 deteainam nor Pe i ratonale ot 2 Exempla, $3 9 sn itl vem THxemplns Sie efectuess etal (—2):3. Avem. ‘Trebate rofinnt i mamere tres ‘Aven Teqults proprietaten cave araténrmitoarae: eit nto erce nuit ‘Tafenel §'f ene ave gumar, adi ‘Orcore ay fi mumard rion a avers Aven Reals pop i cia ttre ti ore Daw oie ror ‘eb numa ai ‘rit ff umd a Sn oven Avot Regula propretaten are anata urmatoanle: eit final 3 ca weal numa rion Oricre wy (8 numa retinal a oem (=f) =e ‘Ave eo Kesuta rations, ni Orisare ae fi nuondralrapanal a, aierit 1.9 orie Avem ReMi pivpristatea care amt urmitou sume roonal, dilrit de very este 0, Oie=u, ident urmatoarle douk proprietti fn care Amparo intre numere rationales nn "VOM # operatia deo M Orica ar fi numeral relimale eb gene €# 0, dae a= by finial se) fy dareg Alte sous, da inpistin en ase noma ei, de {ntee numere rejionale, oblinem numese rationale eyes Oricare ar fé named rationale 0, by ex. dy unde e% 0 gi de dagh a= ¥sie “yatunch ete % Tao de a :e-= 0d vom sera gi 24 Aol spes, pea tn be em enitat tatie ame rahonale find i 11, ateuqimea pe orn EU AceEASE Baz » (-2)+ » (a) 9 (4H nom oy 4 re efestnese: 1 1 4 (2, 8) 245 + (252); 1) (2) 53) (405) 3) (20,05). (—100,6}, 6) (209 39) (10 vo (+ 49) 4 ia) 0. 60 (0s 5m (08) 116) — 64) 0.138) = 2)} oot; 7) 0+ 016] sl 108); 68) — oy 05:2 2H (99 [2 2A} 008}. [-10-(106— 265) + 605-2046 +2468 4.2): (-04. 48) are namie ete wish mae: 9) (07 8 P 1) (0259 son (025 fT ot naa a) se ©) (04) sau (0p? Lay sau (Lt © (—i5~ imo —Tom wear) Ota: » 68 7628) (08 4), 7A) Stoel tone alco ear lo poate hn expr 100 oe ame ei Ls (m2), wade 779 50 0 aH nua el aro ar ft maar natal by dae we atunel a ex. wat ieivait din urttonrele pro ale a, ¢ mal mati decit mro gi 9) Ovicate ar fi aumerele raffonale 0,2, ¢, dak «#5 WB 8 ) Oriciwe af fi moments raionile a, by duck ef = 2 atanoh yr do edevir_« unnbtoarsl prop Disa ar fh numersie rafonale ay by ey Wats Bo 0, € 0 4h Lncrare pentru veificarea tng mor cunoytinf do bass » nee 1 (Lato; j a (ee Fase yt A 2), 9) 100-(—037 10) 9108-405; 11) (60 02 (0.017 13) 164.92 O98; HY 204-306-015) 4-2 805 16) — $000 WSR 2 £18 (1A — 10 ro peatra preities olimpiadelor gf 4 all eonedreurt aed soe diva eu . a 1964-1963 — 2 — — 3986 — 3908) Gas wr 8/1985 (enw modifica) B) Flscare intty neterlp 9 ya ote uk re fochl difaitd matp nee N, Se mal Miserle (2 ‘“ ‘See fle sen name 4) St to electue ®) garth at chucra{i? Patel aplioa cle obsorvato la rlraren exeeiinul? ere iat aes ee erie a RADACINA PATRATL cestest entcarrroans Prin pares tursagt a numirain rajonal 2 wate}, ntloge el mat mare nama itreg Mal tie sau gal i's Avot [1 2a eee ah Pata sole: 18 = 4; 28 = 4; 09-9 ote Numral4 esta patra numaralul 2 ‘Spinem eh 4 esto pat petfert. De asemodea, numerele 16, 10 poeta ‘Numeral 2, 15 Binet tal yes fac air at alam o'r ptvta st arama i Ded mutt ata an ower, pes tet fre su pat lf cd nual wat vite fn N°), atone pirat iu are 2. mxDkersa eurnaTy peermey sewn Din tablal stone a0 yede of patent to. ant porate loans ntti sete mira} natural co eovesponde tn coloana a ous ‘Yom apune ef rilicina pirat a namarulyi 4 ee et Et ant! in ps oa aoe ent een? ed Cm: iam aed Ben eee 2 ee i ag ae Syed Moe EY a Lan fie y= mnie Dac muir marek ese prt wand Data, eet numer edcine pri nama naar Pater sori J = 3; 1 = 9. ‘i a enlenlee 2) 4 Vi Dy! FYE yi 9 VE + 598 2: RKoKeIva PATRATK mIsTRLUS RUALKR RATIONAL NENTOATIY stim on 2-—. Soom ch 2 exe ricinn pists a § sexe JE =2. Anstog, vom sri JT ‘Stim eX 05% = 0.25, Spanem oO eat radia pirat ti 028 sl nctiem YOR — 05. Analog, vi sero: 1257 — 1.5695. Spur tm cu estarEdeinn ata hi 1.02 8 rim YTD 4.2 ‘Define Dect wn nur rational nengaro w fte prtal nae Dani taional nenetie%y tunes x se nuestra patra ‘4 enleutoe: 9) Eso fs vor a vom a: nungcss streak 6b artoxoqi: mark soto rns gpa cpg ri LP DT conn ee Tare patel Bd pated fe ute rinse nominate 2: etal se uotnlo8 he thee nome Re re ‘Analog, putem erie #9 & Tn acest eas, = 9— 38 = 0. Analog, 1245 este oypring ates pateaele a dud nymere naturale ccaneutive, Putem serint = 13,85" Numata 3 ete radeon Stra cu aproximalie dno unitate pri lps a nuenalat 13/6, {ir 1 esto riddeina pateata ew apeosimntc feo unitate pn adand ‘Chumiratal 1243. Resta ate in rest tans R= 12.450 a Destemenes, prom sre: 2" 0, stan Yard = Va> YE: ‘Atragem atenjia ai y2-E9# V4 4 ¥5. oye ») ye oye a yr eva 1) YOO — V2 000 = a0yE Aver: {In guneral patem sore YE =I), unde 2€9. Dee putom serie # — ¢ numai daca 220. senor 1) Sie enloloe: 9) YE BSD; by VOTES = (ot nyse, 2) Si se caleutoco ©) 27 —[2), 2 €Q; 09 VG 6: mormrareas [3 Suen Dap nm oe JE ye _ vt _ a “Ve Dam far3 detonstratie tonema Aner: | Dacia 098 > 0 otansi Bxomple, ° er pent Exot sa ve enter } VEi0 oy Varo | i rt ssh eonsieviun unsle valor aprosimative ale hii y% Prin adaoe epee eres tn ine ea eel reprecnta prnuvo tele recall inf ional te yea 9 aceasta tracie eenial cgi pen proctdenl de exteagere « radi vant reprints pinto fact decimal. fit foie nepor trte din 2 sr 3 vem: 2 n= 3th Adoitom of hecareIrcbie rele ifinité neperiodieh. epreia- a tn usar iio 6 oan: S10), M) (3: BP, BY PL TPS » (fs 9 2f 0 Gi Derr 929 ¢ och of 9 Ef wo) (cg 19 #5 10 00H 1) es 16) vn m7) vu V8: VT: 19 VTE: YY yam O7 5: VR, VT; _VBs 0) yOHK,_ any YHOU ‘sy yTFHOG; 20) (TRON, ue Sh ee caealne (et tel socimateexacte ys facd probe) BEG soci ay fs LBs 40 Vos YY cs 6 a Jed: w fF: w|- Si we calle ty 03403-)2; 85) er. » et yaae ) Save caloueas: 36) Sa ee anéen tm ondineererestare namonsn 0) Core wary ste mai mare: 1.73-y% wan 1. 98 (0) Sie afle cl mai mare numty natural, pret perfect, de petro ite core at alba eit uitaior = (1) Sie alle fy alll nok oumaral Ving (x + yt Be pum 62 Sts ile numersle uaa Sao cre tnte numer riot 4 Sa wale col mai mare uma natu la nian este ar reducing #2, parlect, de forme TIO: forma’ $0ee=, pte de pair cite, tind 465) Peatra ce valori ale Tui 2, nomere naturale, numba natal AEE ato patrat pestoot? 6) Sa se alle toate numero naturde, pirate porfecte, de forma (67) Radicina patrats dia produsul » dont numore naturale dstinote ate 77. Sis allo navel Cl slut are probes? 65) Velusnol unui eub esto do 372,268 msi cul. Si ne alle lung a lat cubul 68) Si to reprezinta fiecare dia vemitourels saul weriind cee realele tae lute aecade : Aafe|zen, 14 ¥hcxe7 Vk pofeiven, =U cy cm 40) Ara unui peat ete elk 00,09. Sc aunginen Itai 171) Stind ea aria unui pateut eto egal 1,44 me afl aba 2) Sind erin uo pltat cto ope ou 10004 mPa te afl Jungimes laturii le xf 1) Sine alle 2 di wip) od Oday 0S! . i [as Mei robo 05 80 de lo pagina 1 LLuceare pentru vevtiearea ful unor cunogtinge de bax 1) 84 se ofeutwee. EYE MLE oT 4) a) Yi00 — VST; 0) Yt + YONI, | 1D QA YE wy fTOOO— YR, 5) TRO + 2) Si se caller a) YAR; b) VIR; 6) VIB: ah VFR «) VTE; 9 TRIES; g) YATBBL; uy TOT 8) Si se calle V7; by RIED; «) DRE a) VOI; 6) YOR; 1) y TUN. St feu gi vonficae 4 86 toeneulez eu doi azole exseto: 9) i) VTE srg on heh pro ale exacte. Sis face gi probe 8) Sis colulozs YES ou tei 9) Sa we aleulene (cu dowd ec 4) YOO; by YOURE. Si ww Tad gh proba. iW Si" se calouex a) VIBE; b) yOOT=O9% 8) Sie» caller of Te TLnceare pentru preities olimpladlor yf ator concurs 49); YI Se Enter wae ko ee 2) Sie sat cnr de frm fu 7 wade WEN na 28) Stind ca ced <0, a) 3/8 + [30 | | ale 4) Sh ce mprozinte wemitoarea mulfime enumerind elementele sale: Aate| 26%, |2| 1 aytua ot mim, pooso-]E 4 est vi numero mai mari decit veo gi dacs = > 4 bai esi ae erie aumesle moval evi dest. Segtle of: X tata i ae serie numerde A, B, C oti B) Dac AX = BY = C2, of oe sere namersle A,B, © online restate ‘Laerare ponirw repelaren unor eunotinfe din eapitoeleanterieare 1) Sé we alle & din ie8; b) 2a5 oF03; DHHS: 07 psn2s pets wendy 9 te Bi Ored act esters teil oy Catenrey ole aS Aettcaiieal peng. post Doris et en seen tn ay Sievtalein dar 200" 3 Feces oa yo Sonne 8g mae Be a tne Goa dare CE weeded Oe ee eee sco ents 60) be Gt kro din apres pit at anaes dn non 17) Mao unui ye gol exte 204% din masa acelviagt vu pi co ans mit Heid, Suited 0 lichidal din vas entorette 16 kg oo tle tft eatarete vo! gol get nthe vam pln 9) Sue ealefewe media proportional » numer: a) 291 7800; by 0.72 8) Caro sat vigor) mmarutu) 42? Dar ai bai —2742 MONOAME $1 ROLINOAME 4. moxoun In manvsll de matematics ponte dasa a Va, am utiliza 0 expr do forma a, av somo nna aby pntra'a expen pe ua Inte tumeree nator, dont grtanake eb. A mah oN linn expose do arma 2a, cae a0 mal mo, penta ima profs inte 2 naman nara dock ajay Canoe: find ninety itregi numer reins puto sre ‘te'forma ("2a sau 21) aaa. Vor bps exp cst opel expel a Yr st Sane eyed oui fate at onc. laneande sb meee mre Pin ied, ezm yo pram rae rt fcr ade fcr Bet eae ken Sorrel io tu thonom, imbolal do imal gt parton s ont de hte on ete po eee Brenple Unitus espe eit monoome Be PR ogi reese Wares eye s(-2): #6 &—-(-3), -* HB (=F). ie Ce (tay 20, 2062120), ( 2a8b)0 Produsle in caro facorialtura int asta oe cara ca pate Pentru acest motiy; in moaomal —(—2t)e- = apar pitrar tele oi a sb Preipunind eel dstvn monom Hat inlets 1 nome "atinaey in orcs monom ptm fad proprietai aman name ‘el rationale anne eamutathittea! W asoritiviaten, De nen, plem schimbe crdins Tretoror tru monoth, pantry & grape [R'antos numerals rationale $l nlocu ou producal Tor, te asset nea, pentru & grape lean io Rta ds aedagt fly pnt a see 6 Singura potero‘ une! Itore. Oposul nul tonom yonte i talonit en produsol dintee"tst'acel monomulu, {inl ama te foul 4 opus unai numar este produ! diatee 1 ace aura. 2 ee Datovith celor de mai aus monomel —(—3e}e- 2 poste fi transformant in fol uemator es Fa (INN) 30h: = (an 2 see ite In nnd anmiator eg EP St ayy Sale ie tare nant anni Sau ort ens en ands ronal pte koe dre iat prot, nuns nals, Selo tn cal create! Tri, coir oa ste Bxenple.-Monoansle 1,1, —21%. ete, Sot, a, Wed, —2y tat ua ma emir anal efor cis aon Trou Nenoanse, rie eb od enn, din snmp atari ney ci oat Ba yh 1co0, can fate ie Tet av ra Manon to ogre nd SU ea eget iet rea oe Baempla, Mononl 0 Graal na mann ifr yin rar ow grape ein star i rapr en» ltoay ete sa expan Leder respec Pe sponta Vane tin’ mowome soso. Dad 0 er se a ae Mure na farce int monon, sxpinental acl Ure Ta el acdoreretn mono conor ee eal 0 are giadul ege) ov 2-244 =5. Exempla. re gradal 6 tn raport ou ters ag Bela monom on 2 tn apart ou itera ap gradu An eu arto orice Mert die dee 8 Otice monom Hind expr, ir ears expe avind valor, ara in sara expresior de numer rafionse ee un noah Foon, wom fare operufit eu toonoame age cuit facemn oper eu aumest ‘ijn enor iti monornull: te aol tn enre- nied cme nest egal 0. fa caaul in care unt nonme esta ep ero, produsul clo oud tho= ‘Exempla. Produsu! monosmelor 52% si — 2y este monomul 5:* 2g) a cil forma eatonieh ene 100" rod setoatlmcsodeaae 02a 2 oah a al Pam wat NBD at 8p) 30) Seg Hs) 28 2y)s30) — alto) 1) 2e- (day, 8) Aa (— 25) 3) 1) (—22)-(—2y); B bey (a) ©) Gey (tah Caen 8) —S2ty- (—atyp), 9) £-(—29; 1) 3 ay WY a (—28)-(—2): 19) 29 (9 1a 20-(—Bey) (40%, 19 oe: (“one Inteoevt fort Ue a rm diet des, cre exponent te mae A exposental eve, puterea a ‘monom este monomul considerst. Dack expooentol este un namie naturel al tare dee fy puters ono monom esta wn prods de monoame, in care factor sint egal dee este‘vn mono Brempla, Puts a troia a monopull zy ete (zy care ae fr St te seri mob forms de monoame, ia forma eanonies,urmttoa- rele putert de, monoune DAP 9 Cavs 9 (a 49 ( aa ate preonat de Sotelo. ssa sina el ao et fie IxWEEFIMES EVEL NoNOM CO UN POLSON Fle manor <¥, poinomnt —2Re +2 @ gi produ lr | z sistributivtste «tomlin ak Fe produsl e—y ts) intro monomul « 4 poinomul #— y= Avem (2— 7 at ka ablite yin desfasecaparonteaelr: Desi Dar y= a= 9) = 08 + (tap. Aas (—t)ey In generat Prodaal cai monam ea on potioom ete am polinom inal facie soma tatror ‘monary fear tore ot pol La Tnmalltes nui monom en un _pe suonomn Boer teem Bsempl. eens fal yeyooealis boc creel au fora Dye ah Wale ol aae—y » AED Wt a by aster — Ber + 2) Paty. eee yf — Sey + Tats itparte Ia ane. raul We donate elt impart intr lino 9 Taig Wi meron iy! ele 4h pig pa nt peat etal apes Salve, polnouia ph — Gat i oom a A Ty ain produ Fecare terme al polnamoul 5y? — ety! ay! impart J mcnobal Sy, dubacte Acastn oo eomstata gin mul 32¥° eat diet Un polinam se imparts fe um manom diferit de sr, doc f termen At polinoida' se imparts le monontulconideret “Stel pari intsepoliom sit aouom, Ia ean in eae olnomag 6 tnparte ls mipatm, ese un palinen ie lage ,polinomal C este ¢gl ov pobinonul a impéetit Ia mone Vomvspune ci firm ett stun ead doteinan puioma ue st tate poinonsl si monomul SS te ecu Wi) aa, DU ee One, DE 23-2} 0) Wx Ay $)23: 1) (Gey — a) 23 9) C+ 2a) 22a); (Ba) 10) (2 — 238) 2(—2s4), A) (ety + Say): zy) 19) (uty (yay 18) (Ohatyt — 19 — 0s ye) (ey, 2. Faeton comes Eiecuited.peosatat‘Lnbs Wbabénl 6 palatal ¢ 4 r Metre abet r+ poate fi expsimata yi sol form heb be Mm Me der + th Monomul apar em factor in tat ere potions din mem. Io} tnt al aeetel egal. Spunen ot manok beste Jest canna interme er Dr ge. Tu mera a dos al eeu get onoual Bayar o wingurk dl sor eon oer ermenilr peinowlat dia ua Ss rere bee We sy factor corn soul ote (Ealta-tay een ey seis aul forma =P) (ext = Say), otnun « mouornulst 2ny. Svoateres fn f¥sloe comin & vant monom diate poinarn dat se face ca mt exempll crmator. Considerdmpolisomul de tal son Pentru a ditefmiua un monom eure af pe Ase oki ean, at ay alegre true tre tilor. fn ene potinomult de mals, Herele wamane.tuvuror to peace Teel aT etre a earete ec te tial har sle sonst ee care tnt tomane tatarortarmenor pl Pam fneamol de faa snk» yf au ghy- Deei ry ate Talon Uf se0e Bn fastor commun om taturor temeailor yoko. Nomaul rational 1 poate fr eos in actor comun, desert aceasta revine I tmpépiren twlueoe 48) 55.0 fle yale. potingonl Pay a= te yp pent: stir ind ee, ‘osficienjioe teemenilor potinomulal eu 2, implaire care se poate face Intatdenuna in mulhimon nvmerelor ronal, en ae Si se doseomponit in factor ) Ap yb) ae ay ‘S40 sere form canoniek monoamele 2) —a2- (at) 2) 22}, 2) ~ 22) Sn yan SSE eae Ca ty Caen en ' Byori; Ge ety + (ode by & Oy 3) (ans 19, SE 1) Coane Ee iF 38) a Ga, Fa a 1) i 2eyk aye) 10) (32a Seah gree (cs . i 1) (20%) (— Lone); 1 (— 2) dae ae Sd se serie sub forma de monoame, in forma eanonied, urmatoarele oe toa, ‘: neents ey—(-tay —1ay)-(-daey tte 1) @Fs 2) eens 1) (209 9) 2.4 Go) (on -2y Bb wt sy aed vei ea Rae lene tenner Brea ad Ge peel eae 1) ox ~ Cas +1) 440 ach: 8) 324 (2 —Fa]-(L +I) 29) aa 2 at (tay 29) a) «(9s Fes dats A mcd sty Pie Sap | 3) (Ba) so bayn Sy baa ty SG) ¢ pelea eoneassa ts” 3 ON Ce a dasa reacts ‘ 3 Ray SS, 8 ate a 2 9) Sety:(—2 8): 39) (A — ws) 2 41) $8 16 ale yaloorea polinomulsi 2 HS apie ; 0) 22% y= 2a hy y= 08 40) St alla vance. polis nt Pah = Bat De pent Of--T eet oe tg cco, 108 89 — = Ky = 2)= yi 108) 8 2) =e = 9) 107 We Py) ME + wy 10g) te — 2) — (5) — vs) 2: 110) inh a 13) mit 1 poinon gun wont Pde Hi) a2 2) fe 9) eee) sats 23) (yok Pate de 124) 2n 3b, 138) Se Sy 186) 52 7 128) 8 2a dag) es 13) 1B) 133) be by; 138) oh bobs 13M) «3 ot 135) on) + We 4-9) 188) ate — 1) — Ko — 1h Sa fe deseompand in factor Pale: 198) 2ey" eye = 6 Saye — hath 10) Deb ale alk 4 Oars HB) at (a +b 13) Se atie ch a= 10g 2, Gwe talealone 144) Se conser Aa)=o Mat 40+ 1 Sis caloulese P08) + {—205), ‘Se eonsierh douk numese: pritol yi al doe La prin ag edu a dain gine obfine a rte. Te Al dalle eo edunt ol etka gi w abt al patroea gama. Stag an pr ve manu 1464) Se considera a = 2" gi b Determinajt valoasca do "edevir a totes din we propouii i) ab; b) a> Bj o) ach, 145) Sts terion buen 10 nomial 1 O10. {8) Sion seri naa 2 mm 4s Lena penten evita insur unor cusoytine de bas 4) Sk walle valoatoa Pe) nous e841 pentru = = 1) Sk 48 fle valoares poinomah cotra: 2 = =I, y =0. St se efectucae urmitouree operat monoeme se reduca termenii asemene: 6) Be dy eg) 2h) Te Feet nee cea Sh ae eloctutn inmlftea diate wn inonom gi un polnom 9} Sate = Bab 1h Sh ge descompond in fact p) fe abj rhea aysa) 22 4 2y +25 0) La; 0) Ay 4 2y aerate penew preitien olinpitlor sta altoreonursun wht fulreg consecutive eate 2) a) Sh te arate oh umerle de orn a Fab se divid eu 104 1) Care ste cl ani nase numnde natural de forma «fa gi care av tl. mai mie numero divi 5)" Numirul iatreg nn este divi nici ou 2 nei ew 3, Sb sa shale a (nin 1) a divide eu 2 1 Sind at's al unt nomiratorla fences © x mumitoral footie 2°" gi al equi eelor dowd rouse obtinate Din ae 43) = HSe+ 0 Tuan fat de adunaro, obtinem Gop 0 = 10 412 sylcad distributivita Aplcindni prose de rele Premplal 3. Fie ecustin b+($—$)= eters ; BP +50 ptm gis § $8 sree in maljimen eva Die alt biteler cise che eae eta dairies time = $= 2; 11) dahedy, 1993 = Ff pe —0; 15) —t0rn 0 1H) 24 bd ahi Ghanis em anny saya bak 42, a eds rat $3) 2) 1 os sicayesiiawe eee ati es eke a) M7 taed 0) 0; $7) t+ amo 16=0; 29) ¥ ®) « 8) 4 —— 78) 0) 2 a) 1 a0; 81) sy 88) 5225 80) T= 1; 90) —Or—= 5 91) Br 1: $2) r= 1: $9) TWH Be 41 =O; 4) 1m) 400 $8) MeN) = Me $A) Mz; M8) Sx 4 2at = 2 + ep) 2=1 244, yop 22251) 100) 09) 2 = Fy 109) 6 -i- nm) ete—as, uy tar—n= te 15s; my 26448 Het) tate; 119) ae 2 M6) 34.5 = 5 4.34; M19) 82—S2=0, 15) 2y =5y 43 Mi) Me 1) =He+2)— 5; 118) 2-1 = 22-49; M9) Se 41) 5 +4; 190) fo 44) = Me 49, m2 =B, (0¥ 0); 122) az = 5 (a7 OF 129) ar = 4 x =a, (a * 0) #0); 198) (e* 0; 198) ber =a +8, (a4 0) 1, (04 0}, 120) fay —b = day, (a OF, 13) Sesh 2s, (@# 0); 199) ee —br= 1, (a8); 133) (+h; 139 ay (or Wy 185) es = be 47, (a #0); 198) (ero) 17) 24 © ulmi sevind 128) Si so represints fens dintss wrtoa lementet cate hte een pes c 0, Datel E=(al2 F u @ ar—6=0; J—tzizeQ, =o) 129) Sis roprsinte Tesare dintee urmitoarele mulfimi srind lementelt sale Inte acca Amiel Basle Catel DatslzeN, 26250) Fotr| erie, Meet CSeIe 4; Ha(a| 20%, Kc ai biz] 2 40) Sim represinte finn dintre uemitoaele multimi, seind lementale sale date aealade AmizireQ Ur4l=2; 2 Cmz|2€Q, —iz-8=0); D=(2 Entele2eQ, Sei); Palele Gmtz|zeQ—% x 141) Sama a dood numer inte Sie afle numeral. Cite slat are problema tn = 6) 102 = 5) mae intro mai ma deet —2 este 1.'Si 143) Suma a dova » te solu aro problema? afl nomeree 148) Se considers nometele rationale a xi b. Stim ei a= b= 0 fee afl care esta mak rare inte | |g 148) entra ce numero tate 2,72 ete 148) Pentrucenumore naturale, numirel ste mom int U8) SH se afl valoaren de adevie a fiedria dintro urmitonrl propor a) te]26Q [21=m = (25a; Dixlze@ [2] =o; = (0) oO (e12EQ, |x| =—5) =; a) L126 Q [able Tierave pentru yeetiearea suitt nor eunatine de tard Si ae rezlve, in mulimea @, cena 8) 320; W RPA; «r=; a) = @) {62 =0; g)2y41~ a by fey 7 2043; 10 = he ))2e—2 moe 0; W) e+) yas St Naebim ast et tym) Ie —2a2thime—sat, oe pirthiernat 1) Sh m serv scvalia cu neeunoscuta dar = (440) 1) She alle wn nome tind 8 adustnd abl siv eu 2 obtinem 2 Recalare. Notind ox x numtralefutat,avem 2x42 a2. Deak z= 22 oy te —4, Prin urmare, 2 Nona ri cantat ete 4 9) Sh oe afte on tude ind of tnmulinde} ou 3 bine noalag reaultat ea tine cindscidem 24 Aestivere. Netind. ou = auratral 0 Nomirolcitat ste —12 8) inna de CEC ds et mn de 3.60 Sua deport. in'adouthnd ente dei ort mare dev Gen da prime ns. Oromia in Tt les ta CE In ecare tant Reasoar. Sa notin ox sun ine depus Ta CEC: In prima lane 2 svn i dp a Ges nt a pea a CEC eotedonk hank este Dec zh te tad Si er ta vow fama fost dou a CE" sma 4 Tote si nt 235ml de a iar neo ab ati deapa GH leap tbe otra in an toate oie le ts ale eat ede tea tra din pia ails in dons uel inc in eae den sit nk 0 afl eanttah egal de apa. Aline 218 —'f 13h Agta 9 Rent of din prima ties trobuio sb turndin i & doa stl 96 ml do ap afl inet in ele dou stele a alle sattli wane de ap ezlar $4 not cus vit nan tai. Aton 92 reed Deei z+ (x — 8) = 60, Desi Da 8 = Od unite 2: ore sau 2768, Prinrmare« ='94 Visa tah ete dele a - a 1) Un tumdr eater 2 mai mate dott sft, $4 9¢afle numerele 1 24 aj mare deet alt numa. Si se ale agete 2) autre 2 » that A zuma lor este 42: fe inde nmallindt ex 2 obtinem.acelas 4) Si 90 atl un feaultat ca atunel cid it adunim ot 24 1) Si se alle un mma lind o8 samaltindat ou 2 abyinem acels eratiat ca atngeh ind seaden pein el tam glodit le wn woe, Lean adunat ea Rozutatal bam inimulf eu2- Din moul rez nm slat 8 any ebfinat nu fa Te-core sia inte Late node m-am dit? 47) Diferontadintre hnginien si linen uni draplnnghi este de 28mm lar perimetul ete de 40, Sts ale arin. dropton shin 5) Soma a tre nutes este 28, Primal este de 3 ort mak mare dete Al dota, ir diferente dintse al deilen i nl treloe este 12. 84 se humerele, £8) Soma bazelor wai rapes este de 25m, far aren lr de 12 m, Site alle beans tnpezel 107) Mia ti spre Ini Petre: Dae i dai fu din ani 1, oi aven do dow oni mat uli baal dct yok aven tw Pete & spane Inv Mi: Bact tnt da tlt sen ite ment Ce sm fre Pere gcse Mia? 11) Sima 4 tei numere tatregi consseutive ee 94, Sh 6 afl name fel (Tol mumnere integt consecutive aint #4, %, 2+), 12) Sumas doul numere natusaly este 257. Dacdsl tmpitien pe nl in ele a elit, ohjinem cit 20 9 restol 5. St se ate Rae (ra mak eelaneinmyan ¢oame we Vigra cu tine end sehen 24-dnl 14) Si se afle un nome stiind eA, inmuljndu:l cu =, objinem ace. 18) Dae din bla) ni sume poxitiy aeilem jomitae pom 1 St a alle su 18) Suma besslor unui trapee eto de 62 tn. Sumstaten uneia este ID mrimat mare doef dublol cuit, Sts alle Bazele fre peut 17) Suma 8 douk nme inte distinete cae ny stat mh, marl ect {ote ‘St afl muerte Cita oll ane peoblens? 16) Un plone a depos In CEC In trek Tank sna, de 620 ei fa a a doa a depor'de dood ori al uf ban det in prima hn fio 20 it mah na tria Ta Ce sm depts Hisar 19) Si ee ale dont momere a owcntive atiind of, dae sain © din unl din le eu din ost, objinem 47. (Dou rumere naturale consecutive sata, # +1) 29) Iatesun erp de cop sot dered ok mal unalté baigdett foto Duh aaa yeni Ia grap Tote plea 7 Dae, atte Numa ral fetelor a fv egal eu > din nymdul bier. Cite fete a lt 21) Tatean Dae din cnr bon sar fr rnin ptr or mal al Hts ty isco Biden tefl de toot ont mai mu ici eet alt eta 4 yaa a rim Bion ich ot al dalle, C1 28) Tntran vas sat 169 ite do ih isi, eae Tok Dap ete regu In prima van vu Macca cantata de hide ha dn va a ovis ramers naturale sevise tn za 40 este galt ‘ou ‘fen Ul din cle ge termi cu nro Teh Kale act ac, tote mumerole naturale de tie fe seis in Dea 10 filed eX fieare intel ndeplingte oars cbndii 1) esto divisiil eu 2 gin este divibil eu 5 steal 3 anne Proweme supimentare 25) Un dreplanghi ate hungimen de tv oot mai mae desi mea Dac et hngen on 2 ai totem 3 in Xin dreptungh « etr-arie ete oy 282 m= mi man dell aa, palit freptanght. St senile Tungimen i atime prima Ereptunghi Un muncitor trebnie x ack; dopa plan pnd Jn» anumit dtd tinue de plese. Pontes abnsia tree ok fh 40 pew po Introdueind @ inovatie, mmeitorl a Tew ele 50 pice pe fi terminate eaiat naar pani do pies ow 3 nal evtermen. Cite piewewveu de fae ibuncitoral dap plat 37) O chips do tractorqti tsbuie xi ae, tran anumit interval do ip, un anvmit ama de eetare, Back ant tha ‘pe Bi, ramise in nema ew 10 hectare fothde plan: Due atte 70 ha eA ene ple tie alle plea pat 28) Un monitor aven de excenta, dup pla, in 14 rly om gnoit nimir de pies, Lucid ite 2 ple yo play fo de plan aenerutatplanul tn 10 ail Sis ae ete poe tron sexe ate in total, mueitora dup plan 29) Un mohil » poreunsdisanta de 166 km ta 2 one. D parte din tft arena de 0 ar lo ir Ae dona ot al are. Cit timp 0 mize mobil event {0 ms tei timp ta migea on esl vite? 80) Distant dintro dous localities de 270 kn, Don utoturi ee ny sel tmp suate Tata man nc sl soininne dupa om don plecare, Sa so ale vier feeraln inte eee dot autataram 51) Doi hits yorneso tm acta timp din Toeaitaten 4 sb were pre lcsitaten 2. vital bila itera de 1 kp We mdoilea are itera de 1 kaw, Pra oj to 0 ce Tnaigten celui deat dale. Care este distanfa Wintee cle ua alta mn. oar hg up upc a data de 30 m de Sisiara guru este de 2 mc Siitura yup sted ne E Limp in save vulpes fae 2 satu, ogaval face 2 sib Aigngatrebuie Al parvurgs gana pentru a ajunge vuln $32) Un droptunghi are fonginen vem mai mare deel ature una ltet,frfationea eu 2 etn mal mica devi lator aera! pata $4) xia a deoptonghi st un pitt atte inh Dacia dreptunghi she Sen ci aria prota Kenge depth fe eu xn nk mare eit atu? Etats, ia ftimen repli hfe ea 2 ean wal mich Ait latuen pte 18) Este posibil sieht 6) Difeenta distre 1m nse natiral gi dublal sit ex 1) Se oe fle arn 2) Dat a> 0, prublere are voli ) Dae a = 0, problema ate st 4 Dock «0, problema are slat 37) O uring & tint tw € lle p tone de 9 pest Der £2 £88) Soma a dont mere ese , ie dileonta dina primal ial doen 40) Cte ilograoe de mnt ow pda dean be Alograma tn eet te eter a a 41) Dowi woine trednien 98 produes improv dip plans term anal ot 3, ean pon 5¢5 tag 42) Un dicta parourge.o anwmitn distant ta 29 yin, BL merge imitate dat distant ew overs. iar calle ou profiad strung 203, 408, 9 $08; tnd *) f=3, 9 fas; pints ms—% 224, p tnt, bter Hae minh ten o pati we BPG kg de 6) 6 muneitori pot termina ohurat in 2 ore, In ete oe pot termina sceeas Iyeraye 10; munete sume 2 i St se fle us nuérsind odin eae wc 11) Si se serie wwmstoarslo aumere in ondine ereseqtoare: 12) Sim wfectuen 2 4(— a) 10 f(a 12) St we efector 107 Tuerare pentru reaizaren recpltaii tinale una easing de ns 1) Namirol 29 este aumir pri? 2) Num] 8 240s divi en 2? Dar on 2 Dat ow 402 Dar ou 22 Daren 82 Dar ou 72 Dav ew 18? Dar 0 1) Si to cfetueen aa a 2 8) Sa ae alle x din o(5—z-aeh BNERCITH §1 PROMLEME DIVERSE SI RECAPITULATIVE, 14) 58 eae THIS ou tri secimale acto ako nc pol 1) Raper doak namere nalarle ese! jar mia Yor ma este | 1) Si so rele stare evn eee, 0 (ett) SMRipeal dor sities retarik is, sa arr Se ale merle, Cle slut are poche jos, atl 5) doos solu 2) o sala (1) Deed famlin yn noms ox soelas renal IV) Nomerle aint avers pro 2a (8) Soma numertor este un aultp de & 4} Ror ines nomeroa «10 cale ae rapa daze he svnn inte nema 3 9 aura os cent ©) SA cousiderim, unitorl gtablow de nomen ' af ee | aaa 12) 4755 - 66 664 — 4.755. 66 666, ese 18) 1.26763 457 4 1) — 1.2 | »|-3= “4 1) 6 6seq2-22 i cc 2 ; 45) 8 2108 4-987 i +987 Fb, ol=f ails oft 4 olf th er eS cd cael cece ek ace ee ie ie Ree ea aaa Dart Oem o a cpus: vid o bucata de hietin de ouloare alba 91 Ue ‘ye 5) = min(o; — 3) “ERLE poral ty a 3 | Seca ese cas eae w(-3pfbed H) Sind che ——29 bb —A eal 10) Sa_se calonleze (la pag. unde 21) So gtie ea 2 Cu eit est a oe 23) Siw al elon de aleve 8 in stoner pe 2) Ovicar arf nuinerels rationale « gif, daot a # 035 44 0 28) Se considers tri mumere positive 2 y= astel Ini 2) Suse ale Gee teei numero stind 24) Se considers tri mumene positive 25 9, = asta inl Cit ta mutt represints nomdral cel mal aio din soma ecw by Sis alee 25) 5a walle aumerele oritive 2, y, 5 £ pind of elo tudeplines In acelagiGamp, urmitoavele condi yb barat ind Snaaltin po 7 00 6) ete gal en di y ear inst numere natrale, primes mal fs este incl diferent be 269) Si om arate din 1) Se dau 183 de momere. Fiecare din ele te sat “1. Se et impart acaste LI) de tment dat goape asta Ine iv pron gro a is egal tau punerlor 29) fn cole oe mrmesel, «sib: int numero. Intro,” Consersen Twxntorul side notre. Priel nmi din reno et fl doien fle egal cu bral tlen ete gal ew difereapa, Ins al doin i pul, al patruln ate egalton dierenta tte al freien iH doiea snd: Ge numa se afl ooul 184 29) Langimen unui droptungh este detrei or nak mae dace 1 rinay arin a ere de 89600 me. Sa ae ale Longin 91 dreptught 30) Roaportal dite tp “2.070 me, Sse ale angina lime dept A) Se conse. Pla) = Bet — 4p 41. Sa se calle 2) FO; 0) 4); 0) AO, $8) S omiderd: ds — 2941, Ba! x Oy, Caz +02 A= BC; AIOE + AC S53) so oral od exbih un polinom P ex coef Pay + Pu = a B 34) Se considera express Bla, ) = — 24), 35) Se considera 84 wp fly valoarea i patra 38) Se consina expres Fl P= WSs — 2fh0y + 2ate~ 29h Rte ‘Ste aflh valoaten expres ponte “ B= Ay 57) 5 gine 84) Si te alle ace non eh oe 1340, 2 7p 1s co ia an a 39) Se considera Uni aumere intvg. Dirt nt primal al Teal cu cierenje Inte ol doi gi a Ulan: Sid He, — 0) + a — 63) + Xe — 0 en 0 49) Si we a ie 4 fvchia din urmitouele poe 9) (al 26%, 244 i Dials 1 o)feleene, + =o) { } afr res o} ei fel u} Dele 7 aie 11) $3 co alle dows annie Intyg rnsnentive sAind oh diferente inten pou ory pleat tale tal me dni ee ele 2 9 by nine nttaral tin wPuthrele lam ne ea pela Toes ebfiaem num da tty le mal ile eal pete Sin deal dat teadem mom Torta din adslay revise A orig Inver, ubjinem,nemarol 9 ema uméeol. Ci soul ae problea? 47) Apa dave sont av, Cite kg de apt obiquuta Ubu Hoge ap de mare, pentru da cet oe OB 1 mumerots paginle wat volon, on tp 50) Intean sae vate 0 canta ou 2 din euntta Bt} trl ale eineva. chelleyle oak le baci, tn prime. ob. te reat de 295 I 58) Cea chattel cn $0 mat et det 1 ain taunt po care 0 aes A dons ai cbeliviegte eu 25 lei mal putin ecit din eat, A toia a chettiegte rota de 295 lel, Ce uma tit i Beearo i? 139%) Mihai « porticipt la un concurs geolr de shiur. Coleg aw treat coe oe alt ce fsa lspns Mihai: Doc jum me din bei are m-a dpa orf pare dst ten ‘RI bisor cre mar f ntwet. Ln concurs a prepa ie baie Cee «cua. ial 1) Intro casi sit de doo ori mai mule fete delt bush a) Pot fn cis 38 elvi? b) Dae 32 50!) Tot ay ast 28 ani gi fil 8, SA se rlaponds a urmaoareé intra ) Poste citi ani vista tata va fide tv ovt mai mare deat lst fila? Dy Belo posible, pe vitor, wat total st fie do 9.5 ort mai Imare dest vista fini? sed esta posibil, aralaji dupa ol ini vinta tatui-va fr de 35 off mal mare’declt viet. fis in vitor, vista atalot 8 fe de patra or ‘ai more dedi vista flui? Dues este posi, aratatt dupa ity ani vis total va fide paten oe anal mare dealt vata i) fost posiil ea, tm treo, viesta ttt 6 fot de patra rit ona fini? Daca. aest Iu fost pen Ii vata‘ ef anim aota ea alee scat ee 50°) So dun segment de drvapté. AB caro are Imgimes de 40 em lig, VITLL) Pe acest segment so eonsidert un punct roi 4 oS ct EE sien, Fie x lungimen tol Ai. Constavim de aceeah parte a hal AB pitas ACDM gi MERE de ltusi AM 3 WB. Fe Y=AC+ CD} DE+EF +B 4 BA. 2) SE ge arate of avem: y—i2e+ aan 2) Si se determine x atl Salty w trulit pitretulul ACDM. Bi) Intro lealiatle yi 2 ate o distant de 290 km, Din A peues Jn acolagi timp eltre B dowd ttn, ee mer pe acelayh dram, ‘Una merge eu vitaze ue 4 anh alla itn do n/t ‘ups cit timp, dela plecares din Ast ainte de a njunge in sand din agin va fi be o dita fata de Be dou on mal mie ect eal resin cu abel poxinee 458) Dowd cumioane ax port sinultn din cate B.. Primal ‘morse vite de 5) kmh jomitate din Un sae « pater Aruna, ier im aestol impulok mers ou 40 ka A doles ex tion a! mers, prima jumitate da drven ea 40 kam iar a dows jumtste ou 20 kf. Care din cele dovd camionne a ajons mat Inti in 504) Un mobil pareonge. jomatate‘dintro dist on wviteea de 40 anf gf exalts junratato eu vitau se GD ka, Care et vitean mes? 60") Soma tri nomore natorale esto 222. Al doles este mai mare ect prin ar rea ete uy -mat mare deel svn, clr Tage ous. Ske ala unter. Cite voli are probleme? 614) Derpre tuck momere stim urmitoxees 4) primal eate: numir natural 2) al doses est musi mate deci’ primal 3) al tele este de dou or mai mate det prim; 4) difernta intr al doilsa gi mul esta mai mare dest 35, 5) mama lor esto 6 97 Si oo ale numeree, Cite soil aro problema? (62) Sa so erate 65 dacd numerele naturale x,y, (y= 2), dt ast felt af Lm sy, atonch y= 2 ‘in probleme date ta otimpiale fn tare noastsa Mia Vie he Vita) avimetiea 3 alee 1072, febrmaie, Bacar Ss caleeon media avitmetics 9! meio geometried (po portional} ‘humereon 2) Langanes unui sgment esto on 175 m mak ied dectt long 2) Sh alle ie $3 tind ci ZaB este un mumie de tii cite diviiil eu 9 fetid ick sailor 4) Sim afl tnt proportional’ egal cu erochil 1173, fauna, Bucuresti p 1) Troi levi au impreank 225 tei. Primal aro 2 din elt are al oven Sr at doit ane 2 in elt ave a trie Ele fe 2) Sane rat frac + contibyind eure care con total 130i 1) Lichidol turnat tntrcun vas clntéreste 153 ke, Vasul plin_cin toot tlt aad Ul Sind vaya ate vlumal de 17 Bh Se nad Steuer pin 9 dentate Hebb 2).0 sum do bani este tmprtith Io 7 levi asta rig peiovesa.jumitace in nt tach aD bani, a ele fue nana var 3 oni al ee Fee jumtate din nol rest inca 3 bani tage mi deporte. ye Selina pret mate de alien at eh fT rg a but, ae acm primi. care 3) Drum parours Sn alunecanen sui erp yom plan inelinat fate proportional ew patrata! simpatit nectar Blancas Un lone In © sound 0 met. Sim lve dura pares acl Earp ‘dae aluneet tin de 8 Soran, ‘4 Dona corpus te mize iniform pe wn een Ele plea in Jag tin dite punet depo ern In consi ontiare, up au tase tated panel #2 finaly cory a al te fend ex x ajung In, iat Sebi eal Gotu, contin. ters Sa aula frou 9 see apne int corpari 195, feraarie, Bucweli 4) Un avtomobil a mers 20 kan eu viet di 400 'tm eu vteet de 60 km (Cae a fot iter mee 3) 4) Care ena a nal mic num natural ma mare det 4474 si cae. enio prs. peront? 3) Cary ete cel. mo ip urge garam mao fi camera parapet tml do 6? gi cave ste pitt poet lip do 7 4974, mai, Bacuestt 4) Un mobil migelnduaswnilorn reeiliy pareurgs 0_ an i 4976 2'kinjh, mobikal parcurge sceedsi Sistanfafuteun timp de dow ort Site ale vtees mobitu. ‘Avila teal plieate tat direot proporionslé eu numeral 1, 46, 25, as diferent fate eal tat mar) el mal sie pert tte de 46 m, te: aile lature fioaoui pat 3) 0 eumi do bani ea itibnit cu primi ta) mmuneiton A, B, 6 alveet proportional ew nuanerele nul din munoitariconstata ci el primegte axtel ex 462 lei saai mint deci dach senasl uma ar fl uistebuil vers {ional eu merce 12, 10 9115 respect eee ne ee ee pet tevin to ordine invest). SA ws arate cd aveest sums , Bae etn Hocars din ei 3 muneitori A, B, C (veriiare). ar enlo ln acest caz cn mai. mare mma ce se poate objine? 9, teraarie, Bucare i apa pe ard 2086 — Arionticd — alg 4257, St wo afle at a 1H, O bunied are dai nepoti.Virta bunict fe exprind priatean | fo dou cite, ficou elit ind vir 5) Stoo afle Lote nomrdle naturale m qin care sitisfac min + 1) = 80. de 83 de ani. Co virsta are bunica? : ae ae destteem, froctia (2) este mai mare docit 2 by In amui 1976 productia unei usine « fyst de A. tone 4) Ginera. spon “Gindegte-te hun nude. Un col inal da ined pe tit. De ta trade the meh primey 30. Inusatijesta terial.” Rettale gat partes av. Tea nimas 252" De Un nomte are proprietaten ef susna intro ia suteloe cea a untae este egal ow eifre tele. Facer. cums dlnee el gt inversatal au (adios numarol format dia aceloei ce [RASPUNSURD gt INDICATIE ato 5) Bs, bt 35) tot. 3m. 1 18) 18) p94) “88h “Gap —2) 058) 08g 5; DTH) “By DAHA 92HSH2H-17 0 HO HI x 4 6 M8. 0,0)0.790.8) 8.92 —B 19 —7. 8 19 8.40 =7 10 Inpities ited, caprronu ¥ oweae napa, ata pe. 18)

You might also like